Die Menschen werden noch für sehr, sehr lange Zeit nicht Gefahr laufen, jemals in (beziehungsweise auf) ein schwarzes Loch zu fallen. Dafür sind diese Himmelsobjekte einfach zu weit entfernt. Schwarze Löcher sind ehemalige Sterne und genau wie diese sind sie einfach zu weit entfernt. Um ein schwarzes Loch zu erreichen und damit zusammenstoßen zu können, muss man erst eine praktikable Methode der interstellaren Raumfahrt entwickeln und das wird noch für lange Zeit, wenn nicht sogar für immer, unmöglich sein. Wir müssten uns also keine Gedanken machen, was mit uns passiert, wenn uns so ein Objekt begegnet (und nein, auch hier auf der Erde werden so schnell keine gefährlichen schwarzen Löcher produziert werden. Aber schwarze Löcher sind nun einmal enorm faszinierend. Wie kaum ein anderes Himmelsobjekt regen sie die Fantasie der Menschen an (wenn auch oft auf die falsche Art und Weise). Und wenn wir an Löcher denken, dann wollen wir natürlich wissen, was passiert, wenn wir reinfallen…

Stephen Hawking hat darüber in seinem berühmten Buch „Eine kurze Geschichte der Zeit“ geschrieben und das Konzept bekannt gemacht: Bei der Annäherung an das Loch sorgt die starke Gravitationskraft dafür, dass man „wie ein Spaghetti gestreckt“ wird. Das liegt übrigens nicht an der Gravitationskraft an sich, sondern an der Gezeitenkraft die sie hervorruft. In der Nähe eines schwarzen Lochs wird die Gravitationskraft bei jeder weiteren Annäherung so schnell so viel stärker, dass an Füßen und Kopf unterschiedlich starke Kräfte wirken und einen so auseinander ziehen – wie eine lange Nudel. Spaghettifizierung lautet daher auch der „Fachausdruck“ für das Schicksal eines Menschen, der in ein schwarzes Loch fällt.

Das alles hat übrigens nichts mit dem berühmten „Ereignishorizont“ zu tun. Das ist die Grenze, hinter der die Anziehungskraft des schwarzen Lochs so groß wird, dass nichts mehr entkommen kann. Dabei handelt es sich aber nur eine gedachte Grenze, kein konkretes physikalisches Objekt. Es kommt dabei auf die Perspektive an. Ein Beobachter von außerhalb kann natürlich nicht hinter den Ereignishorizont blicken. Von dort dringt ja nichts nach außen. Die Person, die in das Loch fällt dagegen, würde am Ereignishorizont nichts spezielles bemerken. Die Geschichte mit der Spaghettifizierung beginnt erst später, wenn man dem schwarzen Loch viel näher ist.

Zwei aktuelle wissenschaftliche Arbeiten kommen aber nun zu dem Schluss, dass das nicht stimmen kann. Der Ereignishorizont muss doch ein spezieller Ort sein. Berücksichtigt man quantenmechanische Argumente, dann ähnelt er einer „Feuerwand“ („Firewall“), auf die man bei der Annäherung an das schwarze Loch trifft. Lange bevor man Gefahr läuft, spaghettifiziert zu werden, wird man also schon durch die Feuerwand terminiert…

Tony Padilla von der Uni Nottingham erklärt die neuen Arbeiten in der aktuellen Folge von Sixty Symbols:

96 Gedanken zu „Was passiert, wenn man in ein schwarzes Loch fällt?“
  1. Bei einer richtig ausgeführten Meditation erreicht man ebenfalls eine innere Singularität, was durchaus auch einem Schwarzen Loch gleicht, allerdings im positiven Sinne.

    Was mich natürlich ebenfalls interessiert, was passiert eigentlich beim Durchschreiten eines physikalischen Schwarzen Loches… stimmen hier wirklich alle Hypothesen und Theorien darüber?.

  2. Ich meine mich zu erinnern, dass man bei der Annäherung an ein stellares Schwarzes Loch spaghettifiziert würde, bevor man den Ereignishorizont erreicht (Größenordnung von 10 km), weil da der Schwerkraftgradient am Ereignishorizont steiler ist als der am Ereignishorizont eines Supermassiven Schwarzen Lochs (Größenordnung mehrere AU), wo es einen folglich erst hinter dem Ereignishorizont spaghettifizieren würde.

  3. @Explikianer:
    Ein Schwarzes Loch lässt sich nicht durchschreiten. Wenn man einmal drin ist, kommt man nicht mehr raus.
    Ob Theorien und Hypothesen über Schwarze Löcher wirklich stimmen lässt sich hingegen schwer sagen. Wie Florian im Artikel schrieb, gibt es keines in der Nähe an dem man es experimentell ausprobieren könnte.
    Dass aber „alle Hypothesen und Theorien“ nicht korrekt sein können, ist hingegen klar. Schließlich gibt es solche die sich gegenseitig widersprechen oder die astronomischen Beobachtungen widersprechen (bzw. solche Hypothesen lassen sich leicht aufstellen).

  4. Wo sieht man eigentlich das Problem bei der interstellaren Raumfahrt? Zumindest in einem Fachbuch zur SRT (ab und zu angereichert mit lustigen Spekulationen) las ich, dass ein nuklearer Antrieb uns mit prinzipiell schon heute verfügbarer Technologie (Hier war die Annahme eine Beschleunigung mit steten gemittelten 10 m/s^2, die in Schüben erreichbar sei) innerhalb der Milchstraße und sogar darüber hinaus reisen lassen könnte. Es gäbe dann natürlich zunehmend mit der Entfernung Probleme (Objekte im Weg, Umgebungsstrahlung wird härter), jedoch die Eigenzeit der Reisenden ist bei entfernteren Zielen überraschend deutlich geringer als die sog. Lichtlaufzeit (durch die Zeitdilatation!), weshalb man kein Generationenschiff bräuchte. Beispielsweise käme man in rund 12 Jahren zum anderen Ende der Milchstraße, allerdings würden bei dieser Entfernung Hindernisse und evtl. auch Umgebungsstrahlung schon ein Problem. Lediglich die Reisezeit von der Erde aus gesehen wäre quälend langsam. Und die Strahlungshärte schlägt noch nicht so sehr zu, wenn man nur nach Alpha Centauri oder so reist (allerdings ist der Zeitvorteil durch Dilatation dann auch gering).
    Ich sehe eher politische Probleme (Risiko des nuklearen Materials beim Start, Reise bringt von Erde aus gesehen erst in 100 Jahren bis hin zu 1000den Ergebnisse) gegen die interstellare Raumfahrt sprechen, als prinzipielle technische Probleme.

  5. Hier war die Annahme eine Beschleunigung mit steten gemittelten 10 m/s^2

    Über welche Zeiträume musst du da beschleunigen?
    Und genau da liegt der Knackpunkt. Die Beschleunigung zu erreichen ist nicht das Problem. Aber die Zeiträume, in denen die Beschleunigung wirken muss – und was natürlich damit zusammenhängt – der dazu notwendige Treibstoff, den du mitführen musst.
    Ca. 95% der Masse eines Saturn/Apollo Stacks auf dem Launchpad waren Sprit. Und für jedes Kilo Sprit mehr, welches du mitnehmen willst, brauchst du wieder mehr Sprit um dieses Kilo zu beschleunigen.

  6. Ich kenne diese Geschichte als theoretisches Gedankenspiel. Das aber eben von der Voraussetzung ausgeht: „Wenn ein Raumschiff konstant mit 10 m/s^2 beschleunigen würde“

  7. Gibt es da nicht auch noch ein 2-tes Problem?
    Je näher man der Singularität des SL kommt, desto langsamer vergeht die Zeit (Standpunkt: Aussenstehender). D.h. die Singularität kann man überhaupt nicht erreichen, weil die Zeit stillsteht. Hat sich ein SL erst einmal gebildet, kann die Singularität im Zentrum überhaupt nicht mehr gefüttert werden, weil keine Materie mehr in endlicher Zeit in die Singularität reinfällt? (Was ja nicht heißt, dass das SL nicht trotzdem wachsen kann.)

    Ich gebe zu, das verwirrt mich jedes mal, wenn ich darüber nachdenke.

  8. @Kallewirsch

    Ich habe das so verstanden:

    Für einen Außenstehenden dauert das Überschreiten des Ereignishorizonts schon (theoretisch) beliebig lange, denn bereits da wird die Dopplerverschiebung unendlich groß und das Objekt scheint einzufrieren und verblasst dann am Ereignishorizont, bis es im Rauschen unsichtbar wird.

    Das eigentliche Objekt fällt aber mit Lichtgeschwindigkeit durch den Ereignishorizont durch, und da es nur ein paar AU zum Zentrum eines supermassiven SLs sind, dauert es nur ein paar Minuten, bis es dort ankommt, nur gibt es niemanden, der das sehen oder messen könnte.

    Aus Sicht des Objekts verkürzt sich die Entfernung vom Ereignishorizont zum Zentrum in der Länge wegen der Längenkontraktion bis fast auf 0 und es erreicht nach seiner Eigenzeit gemessen das Zentrum quasi ohne Verzögerung.

  9. @Alderamin
    Es macht also gar keinen Sinn unsere Zeitmessungen mit denen hinter einem Ereignishorizont zu vergleichen. Das wäre so, wie wenn ein Bewohner eines Paralleluniversums die erste Milliardstel Sekunde unseres Universums beschreiben würde: Diese Zeit wäre unendlich lang für ihn.

    Apropos unterschiedliche Zeitläufte:
    Was macht eigentlich ein Doppelherz Computer dessen eines Herz näher an einer starken Gravitationsquelle steht und deswegen halb so schnell läuft?
    Oder ein biologisches System dessen Blutkreislauf mit einer Extremität in einer anderen „Zeitzone“ ist, mal abgesehen von anderen Effekten, die es tötete?

  10. Die Person, die in das Loch fällt dagegen, würde am Ereignishorizont nichts spezielles bemerken

    Disclaimer: Ich bin kein Fachmann, jedenfalls nicht in diesem Zeugs – Denkfehler von mir sind nicht nur möglich, sondern sogar wahrscheinlich.

    Ich gehe davon aus, dass ich sowohl die Annäherung an den Ereignishorizont als auch sein Überschreiten sehr wohl „bemerken“ würde, da ich wegen der lustigen Raumzeit-Verbiegung eine Menge sehr seltsamer Beobachtungen auf diesem Weg machen müsste.

    Der Ereignishorizont ist ja diese „Sphere Of No Return“, in der die Gravitation so stark wirkt, dass selbst ein Photon nicht mehr entkommen kann. Das bedeutet, dass ein Photon, dass sich wegbewegt (also dessen Bewegungsvektor senkrecht auf der Sphäre steht) durch die Gravitation auf eine Bahn um das ganze Gebilde gezwungen wird. (Da ich befürchte, dass der Ereignishorizont keine exakte Kugel sein wird, ist das Ganze vermutlich etwas komplizierter.) Auf der einen Seite des Ereignishorizontes wird also jedes Photon von diesem weggezogen, in das Zentrum hinein, auf der anderen Seite des Horizontes wäre das gesamte Sichtfeld zum Horizont hin verzerrt. Dass ich den Ereignishorizont überschreite, würde ich daran bemerken, dass diese Grenzlinie zwischen dem schwärzesten denkbaren Schwarz und dem Rest des Kosmos genau in der Mitte meiner Sicht wäre. Als weitere wahrnehmbare Eigenschaft sollte sie im Moment des Durchschreitens keine Krümmung haben — vorher hätte ich eine sich rapide vergrößerende schwarze Scheibe, um die herum sich der Kosmos zerrt; diese würde zu einer schwarzen Fläche, die die Hälfte der Sicht einnimmt.

    Nee, das würde ich bemerken! Gewiss eine sehr interessante Erfahrung, die leider den Nachteil hat, dass man niemandem davon erzählen kann… 😉

  11. Überschreiten sehr wohl „bemerken“ würde, da ich wegen der lustigen Raumzeit-Verbiegung eine Menge sehr seltsamer Beobachtungen auf diesem Weg machen müsste.

    Mag mich irren. Aber üblicherweise ist die Aussage „man bemerkt nichts am Ereignishorizont“ so gemeint, dass man an sich selbst nichts bemerkt. Das du seltsame Effekte beobachtest, sei jetzt mal dahingestellt. Aber du selbst fühlst erst mal nichts Spezielles. Also keine Mauer oder so, durch die du irgendwie magisch durchgezogen wirst.

  12. Obwohl.
    Gedankenexperiment.
    Angenommen ich schaffe es, mit einem Ein-Personen Raumschiff 50 Zentimeter vor dem Ereigbnishorizont zum Stillstand zu kommen. Was sehe ich? Ist da eine schwarze Wand vor mir? Was ist, wenn ich meine Hand über den EH ausstrecke (dann müsste es mich ja eigentlich zerreissen).

    (OK. Ich kann nicht vor dem EH zum Stillstand kommen, weil mich der Gravitationsgradient umbringen würde. Korrekt?)

    Das muss doch schon mal jemand simuliert haben, wie die Annäherung an ein SL (realistisch) aussieht. Gibts denn da kein Youtube Video davon? Wo sind die Verschwörer, die sich bei Youtube auskennen, wenn man sie mal braucht? 🙂

  13. @Ralph Ulrich

    Es macht also gar keinen Sinn unsere Zeitmessungen mit denen hinter einem Ereignishorizont zu vergleichen.

    Ja, physikalisch-messtechnisch gesehen macht es eigentlich keinen Sinn, solche Zeitmessungen zu vergleichen. Mir hat die Überlegung allerdings ein wenig über den Widerspruch der Zeitmessungen verschiedener Beobachter weggeholfen.

    Ganz ähnlich ist das Problem bei der Galaxienflucht. Da kann man ja auch nicht sehen, wo eine Galaxie sich aktuell befindet. Alles, was man sieht, ist uraltes, rotverschobenes Licht. Macht es überhaupt Sinn, darüber zu reden, wo die Galaxie jetzt ist, wenn man kein Licht von ihr empfangen kann? Zumindest kann man sich vorstellen, wo sie sein müsste. Und das kann man auch bei einem Objekt, das mit Lichtgeschwindigkeit in ein SL fällt, denn es wird ja nicht langsamer, nur weil sein Licht immer mehr dopplerverschoben wird.

    Kann man das so sagen?

    Was macht eigentlich ein Doppelherz Computer dessen eines Herz näher an einer starken Gravitationsquelle steht und deswegen halb so schnell läuft?

    Abstürzen 🙂 Im doppelten Sinne des Wortes 🙂

  14. Wenn mich recht entsinne, passt Poul Andersons short story „Kyrie“ ungemein gut zum Thema (und zu Florians kleiner ‚Bücher für den Urlaub‘-Serie).

  15. Wie ist das eigentlich mit dem Schwarzschildradius für den Ereignishorizont genau?
    Es gilt doch
      r = 2GM/c².

    Setze ich für M die Sonnenmasse ein, also 2·10^30kg, erhalte ich den etwa 3km-Radius.

    Setze ich jetzt mal die Masse des Universums ein, also zB. M=10^53kg, dann erhalte ich etwa 1.5·10^26m, also etwa 15Mrd Lichtjahre.

    Heißt das jetzt, daß wir quasi in einem schwarzen Loch leben?

    Herzliche Grüße.

  16. Irgendein Physiker ist mal gefragt worden, was er täte, wenn er sterbenskrank wäre. Seine Antwort lautete, dass er, wenn es denn möglich wäre, am liebsten in ein schwarzes Loch fliegen würde, weil er dann die Theorien aus eigener Anschauung überprüfen könnte. Selbst wenn er seine Ergebnisse vermutlich niemandem mehr mitteilen könne…

  17. Setze ich jetzt mal die Masse des Universums ein, also zB. M=10^53kg, dann erhalte ich etwa 1.5·10^26m, also etwa 15Mrd Lichtjahre.

    .
    ..


    Stimmt. Das *ist* merkwürdig.

    Nur wenn die ganze Masse an einem Punkt versammelt wäre.

    Ist sie aber nicht. Wenn sie aber relativ leer ist, würde man
    für den Schwarzschildradius einen wesentlich kleineren Horizont
    erwarten.

    Es *ist* merkwürdig.

  18. Heißt das jetzt, daß wir quasi in einem schwarzen Loch leben?

    Nur wenn die ganze Masse an einem Punkt versammelt wäre

    Laut seiner Rechnung (die ich nicht überprüft habe) muss es nur auf eine Kugel mit dem Radius von 15Mrd Lichtjahren „gepresst“ werden (das war doch die Definition des Schwarzschildradius, oder?)

    Unser Universum hat allerdings einen deutlich größeren Radius wie 15Mrd Lichtjahre

  19. „Bei einer richtig ausgeführten Meditation erreicht man ebenfalls eine innere Singularität, was durchaus auch einem Schwarzen Loch gleicht“

    dann konsumier mal dmt dann wirste mal merken was selbstauflösung ist

  20. @Alderamin

    Kann man das so sagen?

    Ja.

    Ich erkläre das übrigens gerne so:
    Ein Objekt, das in ein schwarzes Loch fällt, überschreitet aus seiner Sicht heraus ganz „normal“ den Ereignishorizont und erreicht dann in endlicher Eigenzeit die Singularität.
    Allerdings können wir als äußere Beobachter die Überschreitung des Horizontes nicht beobachten, weil wir das Objekt überhaupt nicht direkt beobachten können.
    Wir sehen nur das Licht, das es abgestrahlt hat.
    Das ist selbstverständlich beim „Sehen“ bzw. Beobachten immer und überall so. In diesem Spezialfall ist der Unterschied aber eben ziemlich bedeutsam.
    Das Objekt sendet hier nämlich gar nicht für immer Licht aus, sondern nur, bis es den Horizont überschreitet.
    Auf das abgestrahlte Licht wirkt jetzt während seines Weges zu uns auf eine solchen Weise relativistische Einflüsse, dass wir das Objekt „ewig fallend“ bzw. „eingefroren“ sehen.

    Genauer gesagt gilt:
    Wir sehen von einem hineingefallenen Objekt nur noch endlich viele Photonen, nämlich genau die, die es vor der Überschreitung des Horizontes abgestrahlt hat.
    Ein äußerer Beobachter empfängt im Lauf der Zeit aber immer weniger dieser restlichen Photonen, die zudem immer stärker gravitativ rotverschoben sind.
    Die Photonen, die das Objekt in einer Sekunde Eigenzeit abgestrahlt hat, werden für den Beobachter also über einen allmählich immer größer werdenden Zeitbereich verteilt.
    Irgendwann ist beispielsweise eine Sekunde Eigenzeit des Objekts für einen äußeren Beobachter auf tausende Jahre dilatiert.
    Dadurch bekommt man den Eindruck, dass das Objekt beim Einfallen immer stärker verlangsamt wird und schließlich „einfriert“.
    (Und irgendwann verschwindet es ganz, weil die Strahlung so stark rotverschoben wurde, dass selbst die besten Instrumente sie nicht mehr messen können.)

    Das ist jetzt nur zur Ergänzung gedacht. Du hast das schon richtig verstanden und erklärt.

    @SCHWAR_A

    Das hatten wir doch schon mal, oder?
    Dein M=10^53kg ist die Angabe für die sichtbare Masse des beobachtbaren Universum.

    Das Universum „besteht“ allerdings nach heutigem Wissen nur zu 5% aus sichtbarer Masse, dazu kommen 23 % aus Dunkler Materie und 72 % Dunkler Energie.
    Der allergrößte Teil der Masse fällt bei dir also unter den Tisch.

    Das beobachtbare Universum wiederum hat einen Radius von 46Mrd. Lichtjahren.
    https://www.scienceblogs.de/hier-wohnen-drachen/2010/09/wie-gross-ist-das-beobachtbare-universum.php
    Deine Ergebnis von 15Mrd Lichtjahre hat damit also relativ wenig zu tun.

    Außerdem ist der Schwarzschild-Radius der Ereignishorizont für nicht-rotierendene, ungeladene Schwarze Löcher und folgt aus der Schwarzschild-Metrik.
    Das Universum folgt aber eindeutig nicht der Schwarzschild-Metrik. (Sondern vielmehr der Robertson-Walker-Metrik)

    Der Radius, den man erhält, wenn man nur die sichtbare Materie in die Gleichung für den Schwarzschild-Radius einsetzt, ist momentan auch nur völlig zufällig von der selben Größenordnung wie der Radius des beobachtbaren Universums.
    Wenn man diese beiden Werte für ein beliebiges anderes Alter des Universums berechnet, gilt in der Regel nicht einmal das, da besteht dann überhaupt keine Ähnlichkeit der Zahlenwerte mehr.

  21. sowohl Hubbleradius als auch Schwarzschildradius sind Ereignishorizonte. Und sie sind wohl gerade ziemlich gleich, wär mal interessant das sauber zu rechnen ob das Zufall, oder nur in bestimmten Modellen so ist.

  22. @Niels und Aldemarin:

    Ich möchte an dieser Stelle mal sagen, dass ich mich an und für sich hier, auf SB, der Artikel der Autoren und Autorinnen wegen aufhalte.

    Die Kommentare jedoch, die ja eigentlich immer hauptsächlich sachlich sein sollten, lese ich überwiegend deshalb gerne, weil es hier auch Leute wie Euch gibt.

    Thx!

    (und, bevor jetzt wieder über vermeintliche Kritik an Leuten, die sich womöglich angesprochen fühlen könnten, gejammert wird: jaja, ich bin auch nicht immer sachlich …)

  23. @Alderamin
    Gerne. 🙂

    @Schlotti
    Komplimente machen mich immer so verlegen. 😉

    @goldstino
    Zum einen ist der Hubbleradius aber gar kein Ereignishorizont. Warum er das nicht ist, kann man im schon verlinkten Artikel über das Beobachtbare Universum nachlesen.

    Zum anderen hat die Masse innerhalb des Hubble-Radius natürlich eine völlig anderen Wert als den, den die sichtbare Masse des beobachtbaren Universums hat.

    Wenn man lange genug sucht und Dinge ins Verhältnis setzt, die nichts miteinander zu tun haben, kann man praktisch alles „auseinander herleiten“, man findet zwangsläufig unzählige solcher „Zufälle“.
    Z.B. gibt es Menschen, die ernsthaft behaupten, in den Maßen der Giseh-Pyramide seien praktisch alle Naturkonstanten verschlüsselt, nur weil man selbstverständlich die Zahlenwerte vieler Naturkonstanten erhält, wenn man passend gewählte Maße in passend gewählte Formeln einsetzt.
    So etwas ist bedeutungslose Numerologie.
    Man kann etwa auch in den Abmessungen eines herkömmlichen Fahrrades wahnsinnig bedeutende „Entdeckungen“ machen, wenn man nur will.
    https://psiram.com/ge/index.php/Radosophie

  24. Also meiner Ansicht nach ist dieser Artikel schlecht geschrieben… Ich bin zwar nur Physik Student, aber um es kurz zu fassen:

    Wenn man auch nur ansatzweise in die Nähe von einem solchen schwarzen Loch kommen würde, wären die Gravitationskräfte die da wirken so dermaßen stark, dass es die baryonische Materie einfach zerreißt.

    Die Ausdrucksweise vom Autor „wie ein Spaghetti gestreckt“ finde ich wissenschaftlich betrachtet totaler Quatsch. Die Kräfte die da wirken sind so stark, dass man quasi im Bruchteil von weniger als einer Sekunde zerrissen wird.

  25. Wir können ja schon deshalb nicht in einem schwarzen Loch leben, weil es keinen nennenswerten Schwerkraftgradienten gibt. Das Weltall ist, soweit wir es überblicken, geometrisch flach, die Materie sehr gleichmäßig verteilt, und so wird es hinter dem Hubble-Radius auch noch ein gutes Stück weiter gehen. Je näher man hingegen einem Schwarzen Loch kommt, umso mehr nimmt die Schwerkraft auf unermessliche Werte zu, auch noch hinter dem Schwarzschildradius, das ist doch eine völlig andere Situation.

  26. @Chris: “ Die Ausdrucksweise vom Autor „wie ein Spaghetti gestreckt“ finde ich wissenschaftlich betrachtet totaler Quatsch.“

    Richte deine Beschwerde bitte an Stephen Hawking. Wie im Artikel erwähnt, stammt die Aussage von ihm.

  27. @Kallewirsch&Eisentor „Aber die Zeiträume, in denen die Beschleunigung wirken muss – und was natürlich damit zusammenhängt – der dazu notwendige Treibstoff, den du mitführen musst.
    Ca. 95% der Masse eines Saturn/Apollo Stacks auf dem Launchpad waren Sprit. Und für jedes Kilo Sprit mehr, welches du mitnehmen willst, brauchst du wieder mehr Sprit um dieses Kilo zu beschleunigen.“
    Es ging hier aber um einen nuklearen Antrieb. Der liefert schon ~3% (20% bei Fusion, man braucht das Zeug ja nicht zu halten) der Treibstoffmasse als Energie. Das reicht für den Hauptteil der Milchstraße aus (allerdings braucht man aufgrund des Antriebs ein größeres Raumschiff als bisher je gebaut wurde – zugegeben), wenn man allerdings in 25 Jahren an den Rand des sichtbaren Universums will (kein Witz rechnerisch), reicht dann leider auch kein Materie/Antimaterie-Antrieb mehr, also dauert das dann auch von den theoretischen Grenzen her wohl länger. Es gibt zum nuklearen Explosions-Antrieb auch von der NASA grobe Konzeptstudien, die ein solches Raumschiff als sehr teuer, aber theoretisch machbar angeben (die mögliche Reichweite ist bei der NASA lediglich als bis zu Nachbarsternen angegeben).

  28. @Wizzy

    Es ging hier aber um einen nuklearen Antrieb. Der liefert schon ~3% (20% bei Fusion, man braucht das Zeug ja nicht zu halten) der Treibstoffmasse als Energie. Das reicht für den Hauptteil der Milchstraße aus

    Wohl kaum. Die Mindestenergie (mal ganz ohne Raketenformel und relativistische Massenzunahme), die man braucht, um eine Masse M auf, sagen wir, 50% der Lichtgeschwindigkeit zu bringen, ist Ekin = 1/2 M (c/2)^2 = 1/4 * 1/2 Mc^2, d.h. man müsste ein Viertel des Raumschiffs in reine Energie umwandeln, und braucht nochmal die gleiche Menge zum Anhalten (will man, doch, oder?). Von der Rückkehr gar nicht zu reden.

    Und bei 50% c hat man noch keine nennenswerte Zeitdilatation, deshalb dauert der Flug ans andere Ende der Milchstraße auch nach Bordzeit noch ein paar zehntausend Jahre. Das mit dem Nuklearantrieb kann vorne und hinten nicht stimmen. Damit kommt man vielleicht in ein paar hundert Jahren zum nächsten Stern, aber mehr ist nicht drin. Ohne Antimaterieantrieb kommt man nicht weit. Und Antimaterie ist der gefährlichste Stoff, der überhaupt denkbar ist.

  29. @Alderamin: Nein, meiner Ansicht nach ist Deine Rechnung nicht richtig. In dem von mir genannten Beispiel würde 99% des Raumschiffs zu Start aus Treibstoff bestehen, außerdem lässt man ja das gerade verbrannte Material jeweils zurück und nutzt die kinetische Energie dafür nicht (z.B. stößt man das erhitzte Fusionsprodukt nach hinten aus). Das heißt, jeweils der Rest wird beschleunigt. Das geht dann schon ordentlich ab, ich kann Dir das mal in einer freien Minute iterativ bestimmen.

  30. @Wizzy

    Ich hab‘ ja gesagt, das ist nur eine Hausnummer, ohne Raketenformel (die Raketenformel berücksichtigt, dass man den verbrauchten Treibstoff ausstößt), aber die genaueren Berechnungen hat man für den von mir verlinkten Wiki-Artikel angestellt, und danach kommt man vielleicht mit Fusion zu den nächsten Sternen, aber ans andere Ende der Galaxis, no way.

  31. Die Kommentare sind echt der Wahnsinn.

    Aber ist es nicht so, dass wenn Einstein recht hat und nichts schneller als das Licht sein kann, wir eigentlich niemals den Rand der Milchstraße erreichen können? Die Milchstraße hat einen Durchmesser von ~100.000 Lichtjahren.
    Bei der Reise vom Mittelpunkt bis zum Rand benötigt man bei Lichtgeschwindigkeit 50.000 Jahre Bordzeit. Für die Außenstehenden vergeht natürlich viel mehr Zeit.

    Müsste doch eigentlich stimme oder? Korrigiert mich wenn ich falsch liege.

  32. @Gades

    Nein, nein, wenn Du beliebig nahe an die Lichtgeschwindigkeit herankämst (erreichen kannst Du sie nicht), dann wird die Zeitdilatation beliebig groß, d.h. nach Bordzeit könntest Du in Minuten am anderen Ende der Galaxis sein und in Stunden am Rand des sichtbaren Universums. Das Problem ist nur, dass der Energieaufwand in gleichem Maße zunimmt und gegen unendlich geht, wenn die Geschwindigkeit sich der Lichtgeschwindigkeit nähert.

    Wir können heute in Beschleunigern Teilchen auf 0,999999999c beschleunigen, und je mehr Energie wir hineinstecken, desto mehr 9en kommen hinzu, und die Teilchen werden immer massiver, aber eine 1 mit Nullen ist unmöglich.

    Und so massive Objekte wie Raumschiffe lassen sich leider nicht so leicht beschleunigen wie Protonen. Da liegt das eigentliche Problem.

  33. Das hatten wir doch schon mal, oder?
    Dein M=10^53kg ist die Angabe für die sichtbare Masse des beobachtbaren Universum.

    Also genau die Masse, die *nachgewiesen* ist.

    Das Universum „besteht“ allerdings nach heutigem Wissen nur zu 5% aus sichtbarer Masse, dazu kommen 23 % aus Dunkler Materie und 72 % Dunkler Energie.
    Der allergrößte Teil der Masse fällt bei dir also unter den Tisch.

    Das Problem, das hier immer wieder und gerade bei Astrodictium auffällt: Dunkle Materie und Dunkle Energie sind *nicht* nachgewiesen !
    Was wir haben, ist das Problem, dass sich Galaxien zu schnell bewegen. Dieses Problem läßt sich alternativ und genausogut durch TeVeS beschreiben, indem man die ART für äußerst kleine Beschleunigungen modifiziert (Und ja, der Bullet-Cluster läßt sich damit auch erklären). Solange die beiden also nicht die einzige verbleibende Möglichkeit sind, die Beobachtungen zu erklären, gilt es nicht.

    Das beobachtbare Universum wiederum hat einen Radius von 46Mrd. Lichtjahren.

    Falsch. Diese Größe beruht auf den Annahmen (dunkle Materie / dunkle Energie), die verneint werden können.

    Der Radius, den man erhält, wenn man nur die sichtbare Materie in die Gleichung für den Schwarzschild-Radius einsetzt, ist momentan auch nur völlig zufällig von der selben Größenordnung wie der Radius des beobachtbaren Universums.
    Wenn man diese beiden Werte für ein beliebiges anderes Alter des Universums berechnet, gilt in der Regel nicht einmal das, da besteht dann überhaupt keine Ähnlichkeit der Zahlenwerte mehr.

    Du hast etwas übersehen: Sieh Dir mal den Schwarzschildradius an.
    Es gibt da eine schöne Konstante: Die Gravitationskonstante, *die die Kopplung zwischen Masse und Anziehungskraft* beschreibt. Wenn diese sich jedoch zeitlich ändern würde (aus der Konstante wird ein Parameter), fällt Dein Argument zusammen.

    Dein Einwand, dass es „völlig zufällig“ sei, kann ich auch nicht teilen. Wir haben relativ enge Grenzen, was das Weltalter angeht und es gibt kaum irgendwelche Bedingungen, wie groß das Universum *hätte* sein können. Man kann also nicht irgendwelche Parameter aus dem Hut zaubern, um den Betrag nach Gutdünken anzupassen.

    Ich sag auch nicht, dass das jetzt tatsächlich irgendwas zu bedeuten hat, aber die Koinzidenz ist AFAIK wirklich seltsam.

  34. @TSK

    Das hatten wir doch schon mal, oder? Dein M=10^53kg ist die Angabe für die sichtbare Masse des beobachtbaren Universum.

    Also genau die Masse, die *nachgewiesen* ist.

    Nö, diese Masse ist überhaut nicht nachgewiesen.
    Sie wird vielmehr berechnet, in dem man annimmt, dass unser Universum gerade die kritische Dichte hat.
    Diese Dichte multipliziert man mit dem Volumen des beobachtbaren Universums, wobei man dieses Volumen mit Hilfe des Lambda-CDM-Modell (also dem Modell mit dunkler Materie und dunkler Energie) berechnet.
    Das Ergebnis ist die Gesamtmasse des Universums. Jetzt weiß man, dass die sichtbare Masse nur 5% ausmacht.
    5% dieser Gesamtmasse ergeben ganz genau die 10^53kg.

    Der Zahlenwert 10^53kg ist also völlig von unserem momentanen Modell abhängig.
    Mit jedem anderen Modell ergibt sich ein anderer Zahlenwert. Bei Modellen ohne Dunkle Energie sogar völlig andere Zahlenwerte. Da würdest du dann gar keine „Koinzidenz“ mehr feststellen.

    Das beobachtbare Universum wiederum hat einen Radius von 46Mrd. Lichtjahren.

    Falsch. Diese Größe beruht auf den Annahmen (dunkle Materie / dunkle Energie), die verneint werden können.

    Selbst wenn man nicht an dunkle Materie / dunkle Energie glaubt:
    Der Hubble-Radius gibt auch dann natürlich trotzdem nicht den Radius des beobachtbaren Universums an. Warum sollte er?
    Wie gesagt, ihr vergleicht nicht nur Formeln, die nicht miteinander zu tun haben.
    Du musst wie gerade oben erläutert auch noch falsche Zahlen einsetzen. Und selbst dann gibt es nur für ein ganz bestimmtes Universumsalter eine „Koinzidenz“, für alle anderen Zeiten „koinzidiert“ da gar nichts.

    Du hast etwas übersehen: Sieh Dir mal den Schwarzschildradius an.
    Es gibt da eine schöne Konstante: Die Gravitationskonstante, *die die Kopplung zwischen Masse und Anziehungskraft* beschreibt. Wenn diese sich jedoch zeitlich ändern würde (aus der Konstante wird ein Parameter), fällt Dein Argument zusammen.

    Du willst, dass der Schwarzschildradius, den man aus der beobachtbaren Masse des Universums berechnet, immer den selben Wert hat wie der Hubble-Radius.
    Obwohl beide Formeln in keinem einzigen bekannten kosmologischen Modell das selbe Ergebnis liefern. Richtig?

    Den Wert der Gravitationskonstanten zu ändern hilft dir da überhaupt nichts. Diese Konstante steht nämlich in beiden Formeln.
    Beim Hubble-Radius steht sie nämlich in der Formel für den Hubble-Parameter.

    Aber selbst wenn wir mal ein kosmologisches Modell voraussetzen, in dem die Gravitationskonstante in der Formel für den Hubble-Radius nicht vorkommt:
    Dann ändert sich völlig unerklärlich diese Konstante gerade ganz genau so, dass diese Formeln immer den selben Wert ergeben?
    Das erscheint dir sinnvoll, aber dunkle Materie lehnst du ab? Seltsam.

    Eine veränderliche Gravitationskonstante müsste doch auf Planetenbahnen, Sternenstehung, Galaxienentstehung und allgemein auch auf alle auch anderen gravitativen Vorgänge einen unglaublichen Einfluss haben.
    Neutronensterne werden plötzlich instabil, Galaxien und Sonnensysteme zerreißen bzw. fallen ineinander, Planeten werden aus ihren Umlaufbahnen geschleudert oder stürzen in ihre Sonnen …
    So ein Universum würde völlig anders aussehen als jenes, das uns unsere Beobachtungen zeigen.

  35. @TSK

    Niels:

    Nö, diese Masse ist überhaut nicht nachgewiesen.
    Sie wird vielmehr berechnet, in dem man annimmt, dass unser Universum gerade die kritische Dichte hat.

    … die sich wiederum an der Verteilung von Anisotropien in der kosmischen Hintergrundstrahlung mehr oder weniger direkt messen lässt. Wenn das Weltall die kritische Dichte hat, ist es geometrisch flach und die grössten noch miteinander korrelierenden Strukturen aufgrund der Lichtgeschwindigkeit und des Alters des Universums zur Zeit der Ausstrahlung der Hintergrundstrahlung haben einen bestimmten Winkeldurchmesser am Himmel von ca. 1°. Genau das wird gemessen, mit ca. 1% Genauigkeit. Bei positiv oder negativ gekrümmten Universen mit anderen Dichten wären diese Strukturen größer bzw. kleiner. Wollte ich zum Verständnis der Faktenlage noch ergänzen.

  36. @Alderamin:
    „Winkeldurchmesser am Himmel von ca. 1°“

    Wie genau ist dieser Wert? Du sagst 1%; heißt daß jetzt, daß dieser Winkel 1°±0.01° ist?
    Wo finde ich dazu genaue Daten?

    Herzliche Grüße.

  37. @SCHWAR_A

    Den Wert habe ich aus dem Vortrag von Lawrence Krauss, der hier schon des öfteren verlinkt war (googel Youtube mal nach „Lawrence M. Krauss“, „A Universe from Nothing“). Steht, glaube ich, auch in seinem Buch gleichen Namens drin.

    Ich sagte auch nicht, dass der Winkel 1° +/-1% ist, sondern dass der Winkel ca. 1° beträgt -&nbspden exakten Wert weiß ich nicht, müsste man mal eine exaktere Quelle als Krauss‘ Vortrag suchen)&nbsp- und dass die Messung dieses Winkels die Flachheit des Universums (Abweichung von der kritischen Dichte) mit einem Fehler von 1% bestätigt hat. Das wiederum steht hier, Zeile „Total density of the universe“ in der zweiten Tabelle, Spalte „Best fit (WMAP + BAO + H0)“ (sogar +/- 0,55%, wobei allerdings die Messung von WMAP alleine nur 9% Genauigkeit liefert).

  38. @Alderamin:
    Danke. Der Winkel selbst steht leider nirgends.
    Wie wird der denn aus den anderen Daten berechnet?
    Ich dachte, der Winkel ergibt sich mittels statistischer Methoden aus der Verteilung der Fluktuationen über die Bildfläche…
    Könnte dieser Winkel daher eine so große Unsicherheit besitzen, daß er gar nicht veröffentlicht wird?

    Herzliche Grüße.

  39. @Alderamin:
    Interessant ist auch, daß der „Scalar spectral index“ n < 1.0 ist.
    Wenn ich das richtig verstehe, ist das der Brechungsindex des Mediums „Vakuum“.

    Der wird scheinbar immer kleiner („Running of spectral index“), je weiter man von uns weg geht…

    Herzliche Grüße.

  40. @SCHWAR_A

    Der gemessene Winkel ergibt sich so, wie Du sagst, und dessen Messgenauigkeit bestimmt den Fehler der kritischen Dichte.

    Man muss den Winkel aber mit einem errechneten Wert vergleichen, um die Dichte selbst bestimmen zu können. Den Winkel kann man daraus berechnen, wenn ich das richtig verstanden habe, dass die Rekombination der Wasserstoffatome bei einer bestimmten Temperatur von ca. 3000 K erfolgte (wie das bei Wasserstoffplasma eben so ist). Man berechnet dann zunächst das Weltalter, bei dem die Dichte und Temperatur auf diesen Wert abgesunken ist, und das sind ca. 300000 Jahre (folgt, meine ich, aus z = 1100, da die Hintergrundstrahlung eine Temperatur von ca. 2,7 K hat, also 1100 mal längere Wellenlänge als bei 3000 K). Da Regionen, die sich mit mehr als Lichtgeschwindigkeit voneinander entfernt haben, keinerlei Korrelationen haben können, folgt, dass die größten Korrelationen in der Hintergrundstrahlung über eine Strecke von Weltalter * Lichtgeschwindigkeit = 300000 Lichtjahre bestehen können. Dann muss man noch berücksichtigen, dass das Weltall früher viel kleiner war und damit Winkel am Himmel vergrößert erscheinen, das berücksichtigt man über die Winkelentfernung, und die ergibt dann für z = 1100 und 300000 LJ die ca 1°, die man bei einem flachen Universum erwartet. Für andere Dichten erwartet man entsprechend andere Werte.

  41. @Alderamin:
    …vergiß den Eintrag über „Running of spectral index“…
    Das ist (nur) der CMB-Power-Spektrum-Index, der da wohl variiert…

  42. @Alderamin:
    „Das Universum „besteht“ allerdings nach heutigem Wissen nur zu 5% aus sichtbarer Masse, dazu kommen 23 % aus Dunkler Materie und 72 % Dunkler Energie. Der allergrößte Teil der Masse fällt bei dir also unter den Tisch.“

    Die Dunkle Energie wirkt „anti“-gravitativ, hebt also in der Theorie genau den Anteil wieder auf, der das Universum davor bewahrt, mehr Energie-Dichte wirksam zu haben, als die kritische, zumindest im flachen Universum – und wir haben ein flaches Universum.
    Damit hat unser Universum umgerechnet NUR die besagte wirksame Masse von ~10^53 kg, egal wie sie zusammengesetzt ist.

    Ich denke, daß ist durch die Theorie des Standardmodells so konstruiert, und nicht durch Zufall.

    Als Radius muß man natürlich die Entfernung nehmen, bei der das Licht des hypothetischen „Randes“ oder „Horizontes“ ausgesandt wurde, und das ist bei ~15Mrd. Lichtjahren gewesen. Wo ein damals abstrahlendes Objekt heute ist, ist hierbei ohne Belang.

    Zur Metrik:

    Wir betrachten ja hier nicht den sich ausdehnenden Raum, weil wir hier das Universum als ein einzelnes Objekt behandeln, und zwar wie ein schwarzes Loch – innerhalb eines einzelnen Objektes ist nach der Theorie die Ausdehnung des Raumes ja nicht wirksam (die „Rosinen“ bleiben gleichgroß).
    Stattdessen betrachten wir hier ein einziges gravitatives Objekt, dessen viriale Masse in seinem „Zentrum“ liegt und das etwa 10^53 kg schwer ist.

    Herzliche Grüße.

  43. @Alderamin:
    „Du hast ein Zitat von Niels kommentiert, nicht von mir.“

    …sorry, war’n Doppelpost und Du standest darüber – irgendwie falsch gemerkt…

    Du darfst aber auch was dazu sagen, wenn Du magst…
    …vor allem dann, wenn ich vollkommen daneben liege!!

    @Niels:
    „Das Universum „besteht“ allerdings nach heutigem Wissen nur zu 5% aus sichtbarer Masse, dazu kommen 23 % aus Dunkler Materie und 72 % Dunkler Energie. Der allergrößte Teil der Masse fällt bei dir also unter den Tisch.“

    Sieh‘ bitte den fälschlicherweise an Alderamin adressierten Post hier…

    Herzliche Grüße.

  44. @SCHWAR_A

    Du darfst aber auch was dazu sagen, wenn Du magst…
    …vor allem dann, wenn ich vollkommen daneben liege!!

    Na gut, dann sa’g ich mal was. Schlag mal in Wikipedia die Kosmologische Konstante nach.

    Die Einheit der Materiedichte ist (wie nicht anders zu erwarten kg/m³), siehe Definition von ρc. Der Anteil der der kosmologischen Konstante an der kritischen Dichte steht in der Definition ΩΛ = ρvacc, welche einheitslos ist (auch klar, soll ja 1 rauskommen bei genau der kritistschen Dichte). Der Wert für ΩΛ ist 0,7. Das heißt, ρvac zählt positive kg/m³ zur Dichte des Universums: die Vakuumenergie trägt dazu bei, dass leerer Raum eine Masse hat, obwohl sie gleichzeitig (aufgrund ihres Drucks) zu einer gravitativen Abstoßung führt. Gravitation (abstoßende wie anziehende) ist eben etwas anderes als Materiedichte.

    Anschaulich ist mir das auch nicht klar, aber die Formeln sind ziemlich eindeutig.

  45. @SCHWAR_A

    Ich sollte vielleicht noch ergänzen: wenn die Vakuumenergie einen positiven Beitrag zur Masse pro Volumen leistet, muss diese Masse natürlich auch einen positiven Beitrag zur Gravitation leisten, auch wenn die Summenwirkung aufgrund des abstoßend wirkenden negativen Drucks der Vakuumenergie insgesamt abstoßend ist.

  46. @Alderamin
    Ich finde folgende Überlegung bei der Sache mit dem negativen Druck ganz hilfreich:

    Man denkt als Mensch eigentlich immer in den Begriffen der Newtonschen Physik.
    Dort ist die Quelle der Gravitation die Masse.
    Deswegen findet man es ganz selbstverständlich, dass Gravitation immer anziehend sein muss.

    In der ART ist die Quelle bekanntlich der Energie-Impuls-Tensor.
    Intuitiv erscheint auch hier klar, dass auch der Energie-Impuls-Tensor immer nur anziehende Gravitation bewirken kann.
    Tatsächlich war man bis in die 1960er auch davon überzeugt, dass das tatsächlich so sein muss.
    Das nennt man die „Strong energy condition“ (SEC) und es wurde angenommen, dass die in der ART immer erfüllt sein muss. (Bzw. das die Verletzung der SEC „unphysikalisch“ ist und in der Natur nicht auftritt.)
    (So ähnlich, wie man in der klassischen Physik annimmt, dass die Energieerhaltung immer erfüllt sein muss.)

    Wenn man sich die Formeln aber genau anschaut, sieht man, dass die SEC für bestimmte Anordnungen verletzt wird.
    Insbesondere zeigt eine einfache Rechnung, dass skalare Vakuum-Felder mit positivem Potential (wie die kosmologische Konstante, das Higgs-Feld, die Vakuum-Energie, das Inflatonfeld, …) zu einem Energie-Impuls-Tensor führen, der „abstoßende Gravitation“ erzeugt.

    Das findet niemand anschaulich.
    Man hat jetzt aber die Wahl, entweder die SEC aufzugeben oder das Konzept der skalaren Felder. Das war eine Weile lang eine intensiv diskutierte Streitfrage.
    Weil sich skalare Felder in den Quantenfeldtheorien aber als von überragender Bedeutung herausstellten und weil sich das Konzept der kosmologischen Konstanten (die ja im einfachsten Fall ein konstantes Skalarfeld mit positivem Potential beschreibt) als so fruchtbar für die Kosmologie erwiesen hat, ist man sich mittlerweile einig, dass man die SEC aufgeben muss und nicht die skalaren Felder.
    Es gibt aber trotzdem immer noch Forscher, die das anders sehen.

    Wobei es ja auch immer noch völlig unklar ist, wie die kosmologischen Skalarfelder und die quantenphysikalischen Skalarfelder zusammenpassen könnten.

    Deswegen gibt es vielleicht gar keine anschauliche Erklärung für antigravitative Effekte aufgrund einer positiven Vakuum-Energiedichte (bzw. für gravitative Effekte aufgrund einer negativen Vakuum-Energiedichte).
    Das ist vielleicht einfach etwas, für das es in unserem Lebensumfeld keine Analogie gibt.
    Ich habe jedenfalls bisher noch nirgends eine anschauliche Erklärung gelesen.

    @SCHWAR_A
    Alderamin hat dir schon geantwortet.

    Als Radius muß man natürlich die Entfernung nehmen, bei der das Licht des hypothetischen „Randes“ oder „Horizontes“ ausgesandt wurde, und das ist bei ~15Mrd. Lichtjahren gewesen. Wo ein damals abstrahlendes Objekt heute ist, ist hierbei ohne Belang.

    Du vergleichst deinen „Schwarzschildradius“ (den du mit Hilfe einer aus dem Lambda-CDM-Modell abgeleiteten Masse erhälst, wobei dieses Modell aber nicht gleichzeitig korrekt sein kann) mit was genau?

    und wir haben ein flaches Universum

    Du musst dich schon entscheiden.
    Entweder ist das Universum ein schwarzes Loch oder es ist flach. Beides gleichzeitig geht nicht.

    Wir wissen übrigens ziemlich genau, wie es in einem schwarzen Loch aussehen würde und was ein Beobachter dort sieht.
    (Die Region hinter dem Ereignishorizont ist schließlich nicht das Problem. Bei den Problemen geht es nur um die Beschreibung der unmittelbarste Umgebung der Singularität und der Singularität selbst.)

    Unser Universum sieht aber völlig anders aus als es diese Vorhersagen für einen Beobachter in einem schwarzen Loch aussagen.
    Insbesondere das kosmologische Prinzip kann für einen Beobachter hinter dem Ereignishorizont doch eindeutig niemals erfüllt sein.

    Hier wird beispielsweise beschrieben und in einem Video gezeigt, was man sieht, während man sich im Schwarzschild-schwarzen-Loch befindet und auf die Singularität zufällt:
    https://casa.colorado.edu/~ajsh/singularity.html

  47. @Niels

    Na ja, ein bisschen anschaulich ist die Sache mit dem Druck schon: wenn ich positiven Druck erhöhen will, muss ich Arbeit leisten (z.B. eine Luftpumpe betätigen, um einen Reifen aufzublasen). Da die Arbeit in Form von innerer Energie in das komprimierte Medium geht und Energie zu Masse äquivalent ist, nimmt seine Masse und damit Gravitationswirkung zu. Folglich verursacht positiver Druck (im Vergleich zu Null-Druck) eine anziehende Gravitation.

    Negativer Druck „saugt“ hingegen, d.h. ich entziehe dem System Energie (und damit Masse), wenn ich dem negativen Druck nachgebe. Deswegen bewirkt negativer Druck eine abstoßende Gravitation.

    Warum kollabiert das All nicht, wenn der Druck negativ ist? Aus dem gleichen Grund, aus dem ein See nicht expandiert, obwohl der Druck unter Wasser positiv ist. Der Druck ist überall gleich, eine Zustandsänderung wäre nur bei Druckunterschieden möglich. Aber die abstoßende Gravitation bleibt.

    So ungefähr hab‘ ich’s bei Alan Guth verstanden. Man muss sich nur mit dem Gedanken anfreunden, dass das All komplett und homogen von einem negativen Druck ausgefüllt ist. Ist halt einfach so.

    Was mich allerdings wundert:

    Das nennt man die „Strong energy condition“ (SEC) und es wurde angenommen, dass die in der ART immer erfüllt sein muss. […]
    Wenn man sich die Formeln aber genau anschaut, sieht man, dass die SEC für bestimmte Anordnungen verletzt wird.

    Ich dachte, die kosmologische Konstante ist als Integrationskonstante kompatibel mit der ART, wieso müsste man dann die SEC aufgeben? Das hieße ja, die ART und die SEC sind nicht kompatibel, oder?

  48. @Alderamin

    So ungefähr hab‘ ich’s bei Alan Guth verstanden.

    Ja, so hab ich die Sache mit dem negativen Druck auch verstanden.

    dass das All komplett und homogen von einem negativen Druck ausgefüllt ist.

    Find ich persönlich genau wie das Konzept des negativen Drucks aber überhaupt nicht anschaulich. 😉
    Aber mit der Zeit gewöhnt man sich an alles.

    Das hieße ja, die ART und die SEC sind nicht kompatibel, oder?

    Doch, sind sie.
    Das hab ich wirklich etwas unklar formuliert.

    Ich hatte gehofft, der Zusatz
    Bzw. das die Verletzung der SEC „unphysikalisch“ ist und in der Natur nicht auftritt
    macht es klarer.
    Die ART ist auch dann mathematisch konsistent, wenn die SEC verletzt ist.

    Man kann in die einsteinschen Feldgleichungen allen möglichen Blödsinn einsetzen.
    Nur geht man davon aus, dass entweder die Existenz der einzusetzenden Dinge oder die resultierenden Ergebnisse zu verrückt sind, als das sie etwas mit unserem realen Universum zu tun haben könnten.

    Mal an einem anderen Beispiel vorgestellt:
    In den Formeln der speziellen Relativitätstheorie taucht bekanntlich dieser Ausdruck für die Energie auf.
    Bis 1960(?) war allen Leuten intuitiv klar, dass das bedeutet, dass die Geschwindigkeit v nicht größer als die Lichtgeschwindigkeit werden kann.
    Erst dann fiel jemand auf, dass man auch für v größer als c eine Lösung mit positiver Energie finden kann.
    Dazu muss „nur“ die Masse imaginär werden.
    Das war die Geburtsstunde der Taychonen.

    Soweit ich weiß, werden Taychonen trotzdem von der Mehrheit der Forscher als „unphysikalisch“ bzw. als mathematische Möglichkeit, die aber für unser Universum keine Relevanz hat, eingestuft.
    Nicht jede mathematische Lösung einer Gleichung muss auch in unserem Universum „verwirklicht“ sein.

    In der ART stellt man Bedingungen an den Energie-Impuls-Tensor, die zwar mathematisch nicht auszuschließen sind, die man aber für dringend notwendig hält, damit man ein „vernünftiges“ Ergebnis bekommt.
    So eine Bedingung nennt man „Energy condition“.
    Ich hab gerade mal gesucht, dazu gibt es auch einen Wikipedia-Eintrag.
    Vielleicht wird durch das dortige „Motivation“-Kapitel klar, was ich sagen will:
    https://en.wikipedia.org/wiki/Energy_condition#Motivation

    Mittlerweile geht man wie gesagt davon aus, dass die Strong energy condition nicht gültig sein muss.
    An andere Energiebedingungen glaubt man aber immer noch.
    Viele der Aussagen, die man über schwarze Löcher macht, kann man nur aufstellen, wenn bestimme energy conditions gelten.

    Um durchquerbare Wurmlöcher herzustellen, muss beispielsweise die Null energy condition verletzt werden. Deswegen die Skepsis vieler Wissenschaftler bezüglich der Existenz solcher Wurmlöcher.
    (Eine Möglichkeit diese Bedingung zu verletzen liefert die berühmte exotische Materie mit negativer Energiedichte.)

    Auch fast alle Formen der Zeitreise in die Vergangenheit sind nur dann möglich, wenn Energiebedingungen verletzt werden.
    Zeitreisen mag niemand, deswegen würde die Erfüllung dieser energy conditions ne tolle Sache.

    Ohne die Annahme von Bedingungen ist es auch praktisch unmöglich, viele Aussagen über schwarze Löcher aus den Gleichungen der ART herauszuholen.
    Zum Beispiel beruht das No-Hair-Theorem auf der Annahme der Gültigkeit von energy conditions .
    Allgemein kann man nur dann Singularitäten-Theoreme aufstellen oder über die Entropie schwarzer Löcher sinnvolle Aussagen machen, wenn man die Gültigkeit bestimmter Energie-Bedingungen voraussetzt.

    Früher hielt man auch den „negativen Druck“ für so verrückt, dass man sich sicher war, dass so etwas ausgeschlossen werden kann.
    Wie gesagt, mittlerweile hat sich das geändert.
    Mal schauen, ob das bei den anderen Bedingungen auch noch passiert.

    Hm, so viel wollte ich gar nicht schreiben.
    Führt wahrscheinlich viel zu weit, aber ich will es jetzt auch nicht mehr löschen.

  49. @Niels

    Dazu muss „nur“ die Masse imaginär werden.
    Das war die Geburtsstunde der Taychonen.

    Wohl wahr, komplexe Zahlen sind irgendwie schon ein sehr willkürliches Produkt mathematischer Definition: ich tu‘ mal so, als ob -1 eine Wurzel hätte, mal sehen wohin das führt. Erstaunlich, dass sie in der Physik manchmal ganz hilfreich sein können, wie z.B. in der Signalverarbeitung, aber da wohl auch nur, weil sie die Phase einer Trägerwelle als Argumentwinkel abbilden können. Es wäre da aber auch ohne sie gegangen, und jeder andere 2D-Vektor hätte genau so funktioniert.

    Ohne die Annahme von Bedingungen ist es auch praktisch unmöglich, viele Aussagen über schwarze Löcher aus den Gleichungen der ART herauszuholen.
    Zum Beispiel beruht das No-Hair-Theorem auf der Annahme der Gültigkeit von energy conditions .

    Gilt das No-Hair-Theorem überhaupt noch? Ich meine mich zu erinnern, vor nicht allzulanger Zeit gelesen zu haben, das schwarze Löcher vielleicht doch nicht alle Information vernichten und somit doch ein paar Haare haben könnten. Kann mich aber auch irren.

    Hm, so viel wollte ich gar nicht schreiben.
    Führt wahrscheinlich viel zu weit, aber ich will es jetzt auch nicht mehr löschen.

    Danke für den Link auf die Energy Conditions, lese ich mal in Ruhe durch. Och, schreib‘ ruhig ausführlich, wenn Du Dich sonst schon relativ rar machst. Kann man immer was bei lernen. 🙂

  50. @Alderamin

    Wohl wahr, komplexe Zahlen sind irgendwie schon ein sehr willkürliches Produkt mathematischer Definition:
    […] Erstaunlich, dass sie in der Physik manchmal ganz hilfreich sein können

    Ich will jetzt nichts falsches erzählen, aber es gibt doch gar keine Möglichkeit, Quantenphysik ohne Hilbertraum und komplexe Zahlen zu betreiben, oder?

    Gilt das No-Hair-Theorem überhaupt noch?

    Die Mehrzahl der Forscher sieht das jedenfalls so.
    Das war mal recht prominent in den Medien, weil Hawking aufgrund einer eigenen Veröffentlichung von 2004(?) seine Wette über das Schwarze-Loch-Informationsparadoxon (Thorne und Hawking gegen Preskill) als verloren akzeptiere.

    Laut Hawking gilt das No-Hair-Theorem also nicht mehr.

    Die große Mehrheit der Forscher auf diesem Gebiet sieht das aber anders, jedenfalls soweit ich das überlicken kann. Der Konsens ist wohl, dass Hawkings Arbeit solche Schlussfolgerungen nicht hergibt.

    Auch Thorne und sogar Preskill (der Gewinner der Wette) halten das Paradox noch für offen.

    Wobei es hier aber um Probleme bei der Zusammenführung von Quantenphysik und ART geht.
    Da hab ich zum einen echt wenig Ahnung. Ich brauch erst mal noch ein paar Jahrzehnte, bis ich jede der Einzeltheorien tief genug verstehe. Dann sehn wir mal weiter. 😉
    Zum anderen glaube ich, dass hier immer nur vorläufige Ergebnissen drin sind, die sich vermutlich zum großen Teil als sinnlos und völlig falsch erweisen werden, sobald man das Ganze endlich im Rahmen einer Quantengravitation untersuchen kann.

  51. @Alderamin & Niels:
    Zunächst mal vielen Dank!

    Inzwischen habe ich bemerkt, daß man beim „Schwarzen Loch“ immer davon ausgeht, daß es im Inneren eine Singularität gibt. Mir war bisher nur die Singularität des Ereignishorizontes bewußt…

    Es dürfte jedoch m.E. von außen gesehen keinen Unterschied machen, wie die Masse im nicht-rotierenden SL verteilt ist, sie muß lediglich virial zentralsymmetrisch wirken. Damit gilt die Schwarzschild-Gleichung für Abstände größer-gleich den Schwarzschild-Radius auch hier.

    „Du musst dich schon entscheiden. Entweder ist das Universum ein schwarzes Loch oder es ist flach. Beides gleichzeitig geht nicht. “

    Danke, das ist natürlich das Argument schlechthin!
    An notwendigerweise gekrümmte Lichtwege innerhalb meines hypothetischen SL-Universums hatte ich bisher noch nicht gedacht…

    Sehe ich es also richtig:

    Die Dunkle Energie biegt sozusagen die durch die Gesamt-Gravitation erzeugten Lichtweg-Krümmungen wieder zurück, und zwar exakt so, daß das Ergebnis als flach gemessen wird, also: ein Lichtwege-Dreieck behält seine Winkelsumme 180°, egal, welche Orientierung es im Raum aufweist?

    Herzliche Grüße.

  52. @SCHWAR_A

    Kommt darauf an, was Du unter „zurück“ verstehst. Wenn die Masse der dunklen Energie fehlen würde, hätte das Weltall nur 30% der kritischen Dichte und wäre damit offen, d.h. die Winkelsumme im Dreieck wäre kleiner als 180°. Mit der dunklen Energie ist das Weltall flach, die Winkelsumme ist genau gleich 180°. Obwohl sie eine Abstoßung bewirkt.

    Früher, in den 80ern, da war das noch ganz einfach: Wenn ich einen Stein nach oben schmeiße, kommt er normalerweise wegen der Anziehung der Erde wieder zurück. Schmeiße ich ihn mit Fluchtgeschwindigkeit weg, dann wird er immer langsamer und erreicht im Unendlichen eine Geschwindigkeit von 0. Das ist genau der Grenzfall zu dem Fall, dass ich den Stein noch schneller wegwerfe und er das Schwerefeld der Erde mit einem Geschwindigkeitsüberschuss verlässt.

    So galt das nach dem Verständnis in den 80ern für das Universum insgesamt: Es hatte, so dachte man, einen gewissen „Schwung“ beim Urknall erhalten. Wenn dieser nicht groß genug war, würden die Galaxien irgendwann wieder zu einem gemeinsamen Ort zurückstürzen, dann wäre das Weltall geschlossen. Wenn der Schwung groß genug war, die Fluchtgeschwindigkeit zu überschreiten, dann würden die Galaxien auf ewig auseinanderstreben und das Weltall wäre offen. Im Grenzfall wäre das Weltall genau flach und die Expansion würde sich auf ewig mit der Grenzgeschwindigkeit 0 verlangsamen.

    Nun kommt die dunkle Energie dazu und bläst das Weltall auch im flachen Universum mit zunehmender Geschwindigkeit auf, das macht die Sache nicht eben leichter verständlich. Lawrence Krauss druckst sich in seinem Buch ein wenig um diesen Punkt herum. Vielleicht kann man es sich so vorstellen (mein laienhaftes Verständnis):

    Anfangs, als das Universum klein war, war es genau so wie oben beschrieben, die Vakuumenergie spielte noch gar keine Rolle und das Weltall hatte mit der eingeschlossenen Masse (dunkle und gewöhnliche Materie) genau die kritische Dichte und war geometrisch flach. Die Dichte der Materie nahm mit zunehmender Ausdehnung ab, die kritische Dichte im gleichen Maße, denn sie hängt ja davon ab, in welchem Volumen die gravitativ aufeinander wirkende Materie verteilt ist.

    Je mehr sich das All ausdehnte, desto mehr Vakuum mit seiner Energiedichte kam hinzu. Die Energiedichte des Vakuums ist aber konstant, sie ist eine Eigenschaft des leeren Raums, die nicht mit der Expansion des Weltalls abnimmt. Folglich nimmt sie einen immer größeren Anteil der kritischen Dichte an, bis sie im unendlichen Grenzfall 100% der kritischen Dichte ausmachen wird, wenn die Materie dereinst homöopathisch im Weltall verteilt sein wird. Sie muss dabei exakt die kritische Dichte haben, weil ansonsten die Energiebilanz nicht stimmt (nach Krauss heben sich die potenzielle und die Massen-Energie genau zu 0 auf, weil das Weltall aus Nichts entstanden ist, und das ist genau der Fall in einem flachen Universum).

    Das hat aber auch schon während der kosmischen Inflationsphase gegolten, als die gravitative Abstoßung und der negative Druck viel größer waren. Auch da muss die Dichte am Ende genau die kritische gewesen sein (deswegen bemüht man ja die kosmische Inflation als Erklärung des Flachheitsproblems). D.h. die gravitative Abstoßung bzw. die Größe des negativen Drucks spielt letztlich keine Rolle für den Wert der kritischen Dichte oder die Raumkrümmung.

    Nur so scheint das ganze Sinn zu machen. Ein bisschen.

  53. @SCHWAR_A

    Inzwischen habe ich bemerkt, daß man beim „Schwarzen Loch“ immer davon ausgeht, daß es im Inneren eine Singularität gibt. Mir war bisher nur die Singularität des Ereignishorizontes bewußt…

    Die Singularität im Zentrum ist aber die einzige „echte Singularität“.
    Am Horizont befindet sich in Schwarzschild-Koordinaten nur eine sogenannte Koordinatensingularität. Das bedeutet, dass diese Singularität bei geeigneter Koordinatenwahl verschwindet. Damit kann sie auch keine tiefere Bedeutung haben, wie sie die echte Singularität im Zentrum besitzt.
    Schließlich muss es für die Physik egal sein, welches Koordinatensystem wir verwenden.
    Eine Koordinatensingularität zeigt eigentlich nur, dass die gewählten Koordinaten für die genaue Beschreibung der zu untersuchenden Situation ungeeignet sind.
    In unserem Fall wären geeignete Koordinaten zum Beispiel die Kruskal-Szekeres-Koordinaten. In diesem Koordinatensystem tritt am Ereignishorizont keine Singularität auf, da wird also nichts unendlich.

    Koordinatensingularitäten sind überhaupt nichts Mysteriöses oder Ungewöhnliches.
    Zum Beispiel haben ganz normale Polarkoordinaten für den zweidimensionalen euklidischen Raum im Nullpunkt eine solche Singularität.
    Im dreidimensionalen euklidischen Raum weisen Sphärische Koordinaten im Nord-und Südol Koordinatensingularitäten auf und bei Kugelkoordinaten macht die ganze z-Achse Probleme.

    In kartesischen Koordinaten ist an diesen Stellen dagegen nichts Ungewöhnliches zu entdecken.

    Es dürfte jedoch m.E. von außen gesehen keinen Unterschied machen, wie die Masse im nicht-rotierenden SL verteilt ist, sie muß lediglich virial zentralsymmetrisch wirken.

    Keine Ahnung, was du mit „virial zentralsymmetrisch wirkend“ meinst.

    Ich weiß auch nicht, was du hier mit dem Begriff „viriale Masse“ meinst, den du weiter oben verwendest.

    Jedenfalls gilt ja in bekanntlich der der Newtonschen Gravitation, dass das externe Gravitationsfeld einer sphärisch-symmetrischen Massenverteilung gleich dem einer Punktmasse in der Sphärenmitte ist.
    In der ART gilt das verallgemeinert natürlich immer noch und heißt dort Birkhoff-Theorem.
    Aus wiki:
    Eine sphärisch symmetrische Vakuumlösung der einsteinschen Feldgleichungen außerhalb einer Massenverteilung muss statisch sein und diese Lösung muss die Schwarzschild-Lösung sein.
    Konsequenz daraus ist zum Beispiel, dass ein Stern, der perfekt kugelsymmetrisch zu einem schwarzen Loch zusammenfällt, dabei keine Gravitationswellen abstrahlt.
    Auch kugelsymmetrische, pulsierende Sterne strahlen deswegen nicht gravitativ.

    Das bedeutet aber nicht, dass deswegen die Materie hinter dem Ereignishorizont beliebig verteilt sein kann.
    Die Formeln sind da absolut eindeutig.
    Alles, was sich hinter dem Ereignishorizont befindet, muss auf die Zentrums-Singularität zufallen.

  54. @Alderamin

    Nun kommt die dunkle Energie dazu und bläst das Weltall auch im flachen Universum mit zunehmender Geschwindigkeit auf

    Der Witz ist eher, dass mit dunkler Energie auch ein positiv gekrümmtes, sphärisches Universum unendlich lange und sogar beschleunigt expandieren kann.
    Selbst wenn unser Universum also sphärisch und geschlossen ist, kann es ewig expandieren und muss nicht zwangsläufig irgendwann wieder zusammenfallen.

    Die Energiedichte des Vakuums ist aber konstant […]
    Folglich nimmt sie einen immer größeren Anteil der kritischen Dichte an, bis sie im unendlichen Grenzfall 100% der kritischen Dichte ausmachen wird

    Bei einem perfekt flachen Universum ist das so. Sonst nicht.

    Sie muss dabei exakt die kritische Dichte haben, weil ansonsten die Energiebilanz nicht stimmt.
    (nach Krauss heben sich die potenzielle und die Massen-Energie genau zu 0 auf, weil das Weltall aus Nichts entstanden ist, und das ist genau der Fall in einem flachen Universum).

    Na ja.
    Dann stimmt eben die Energiebilanz nicht. Das ist in der ART ausdrücklich erlaubt.

    Allein aus Flachheit lässt sich das auch nicht schließen.
    Flachheit und Null-Energie-Gehalt sind nämlich nicht äquivalent.
    Ein Universum mit einer Gesamtenergie von Null muss flach sein, flache Universen können allerdings problemlos eine Gesamtenergie ungleich Null haben.
    Dass unser Universum ein Null-Energie-Universum ist, ist nur eine Hypothese. Zugegeben eine, die seit Neuestem sehr beliebt ist.
    Niemand weiß, ob unser Universum tatsächlich aus dem Nichts entstanden ist. Wir wissen ja eigentlich nicht einmal, ob es tatsächlich flach ist.

    Die Dichte der Materie nahm mit zunehmender Ausdehnung ab, die kritische Dichte im gleichen Maße, denn sie hängt ja davon ab, in welchem Volumen die gravitativ aufeinander wirkende Materie verteilt ist.

    Nein, gerade nicht in gleichem Maße. Das ist genau das Flachheitsproblem.

    Wenn man die Friedmanngleichung passend umformt, bekommt man

    [1-(ρ/ρc)-Ωlambda)]*H²*a² = -k = konstant

    ρ ist die Materiedichte
    ρc ist die kritische Dichte
    Ωlambda ist der Dichteparamter der dunklen Energie Ωlambda = lambda/(3*H²)
    lambda ist die kosmologische Konstante
    H ist der Hubble-Parameter
    a der Skalenfaktor
    Jede dieser Größen außer k und lambda ist zeitabhängig!
    k der Krümmungsparameter der Robertson-Walker-Metrik

    Wenn man das ein bisschen zusammenfasst, bekommt man
    (1-Ω)*H²*a² = -k = konstant {1}
    Ω ist der Dichteparameter des Universums, also Ω = (ρ/ρc) + Ωlambda

    Jetzt betrachten wir ein junges Universum ohne(!) Inflation.
    Im jungen Universum können wir den Dichteparameter der dunklen Energie weglassen.
    Gleichung {1} vereinfacht sich dadurch zu
    [(1/Ω)-1]*ρ*a² = konstant
    Wobei jetzt Ω = ρ/ρc

    Aufgrund der Konstanz kann man verschiedene Zeitpunkte t und t1 gleichsetzen:
    Also [(1/Ω)-1]*ρ*a² = [(1/Ω{t1}))-1]*ρ{t1}*a{t1}²

    Wir wissen außerdem, dass per Definition für den Skalenfaktor gilt:
    1+z = a(t1)/a
    z ist die Rotverschiebung.

    Außerdem wissen wir, wie die Dichte des Universums vom Skalenfaktor (und damit von z) abhängt.
    In der Materiephase liegt praktisch nur Materie vor, die Dichte sinkt also natürlich proportional zum Volumen.
    ρ ~ 1/(a^3)

    In der Strahlungsphase liegt praktisch nur Strahlung vor, die Dichte sinkt mit dem Volumen, es findet aber gleichzeitig noch Rotverschiebung statt:
    ρ ~ 1/(a^4)

    Wenn man jetzt für die Materiephase ρ = 1/(a^3) und 1+z = a(t1)/a in obige Gleichung einsetzt, bekommt man
    [(1/Ω)-1] = [(1/Ω(heute))-1]/[1+z]
    Ω(heute) ist der heutige Wert des Dichteparameters, also
    Ω(heute) ist größer als 0,95 und kleiner als 1,05.

    Das bedeutet, dass der Dichteparameter Ω für den Zeitpunkt mit der Rotverscheibung z einen Wert
    größer als 1/[1+0,05/(1+z)] und
    kleiner als 1/[1+0,05/(1+z)] haben musste.

    Für die Strahlungsphase muss man in diesen Gleichungen sogar (1+z) durch (1+z)^2 ersetzen.

    Da der Dichteparameter heute größer als 0,95 und kleiner als 1,05 ist, durfte er vom Wert 1

    1. 50 Millionen Jahre nach dem Urknall (z = 1000) nur um plus/minus 0,00005 abweichen
    2. eine Sekunde nach dem Urknall (z=10^10) nur um 0,00000000000005
    3. zur Plankzeit nur um etwa 10^-70

    Damit unser Universum heute also ungefähr flach aussieht, muss es zu Beginn praktisch perfekt flach gewesen sein und es muss praktisch perfekt mit der kritischen Masse entstanden sein.
    Dass das Universum von allen unendlich vielen möglichen Werten für Omega ausgerechnet mit einem so unglaublich extrem dicht am Zahlenwert 1 begonnen hat ist aber nicht besonders befriedigend.
    Das nennt man das Flachheitsproblem.
    In der englischen Wikipedia wird das so ähnlich aber etwas anders erklärt, vielleicht kannst du damit mehr anfangen:
    https://en.wikipedia.org/wiki/Flatness_problem#Energy_density_and_the_Friedmann_equation

    Wenn es wirklich so klar wäre, dass unser Universum aus dem Nichts entstanden wäre und einen Null-Energie-Gehalt hätte, würde sich dieses Flachheitsproblem natürlich in Luft auflösen. Völlig ohne Inflation.
    Der Zahlenwert wäre einfach von Beginn an genau Ω=1 und fertig.

    Die Inflation löst das Problem folgendermaßen:

    Während der Inflation gilt H = Wurzel[lambda_inflation/3] = const

    lambda_inflation ist die (kosmologische) Konstante der Inflation.
    (Friedmanngleichung anschauen. Alle Terme außer diesem können vernachlässigt werden.)

    Wenn man sich jetzt Gleichung {1} anschaut und bedenkt, dass H in sehr guter Näherung konstant bleibt, der Skalenfaktor a aber unglaublich stark anwächst (mindestens um den Faktor 10^30) sieht man folgendes:
    Damit die Gleichung immer noch den selben konstanten Wert annimmt, muss Ω gegen den Wert 1 gehen.
    Das Universum kann vor der Inflation also einen fast beliebigen Wert für Omega gehabt haben.
    Durch die Inflation wurde Omega dann zwangsläufig auf einen Wert, der mit sehr großer Genauigkeit Omega=1 ist, gesetzt.
    Das Universum wurde durch die Inflation praktisch perfekt flach. Dabei ist es fast egal, wie es vorher aussah.

    Nach Abschluss der Inflation kann sich Omega dann wieder von 1 wegentwickeln.
    Allerdings wurde es durch die inflationäre Phase so extrem nahe zu 1 gesetzt, das die bisher vergangene Zeit einfach noch nicht gereicht hat, um sich wieder weit davon wegzubewegen.
    Durch die Inflation sehen also auch hyperbolische und sphärische Universen mit starken Anfangskrümmungen für unser heutiges Universumsalter sehr „flach“ aus.

    Anfangs, als das Universum klein war, war es genau so wie oben beschrieben, die Vakuumenergie spielte noch gar keine Rolle und das Weltall hatte mit der eingeschlossenen Masse (dunkle und gewöhnliche Materie) genau die kritische Dichte und war geometrisch flach.

    Wie gesagt, eine Stärke des Inflationsmodells ist es, dass das gerade nicht so sein muss.

    Das extrem frühe Universum kann eine fast beliebige Krümmung und Dichte gehabt haben.

    Sie muss dabei exakt die kritische Dichte haben[…]
    Das hat aber auch schon während der kosmischen Inflationsphase gegolten, als die gravitative Abstoßung und der negative Druck viel größer waren.

    Das ist jetzt eigentlich ne unnötige Wiederholung, aber egal 😉 :

    Nein, während der Inflationsphase galt das gerade nicht.

    Das hat aber auch schon während der kosmischen Inflationsphase gegolten, als die gravitative Abstoßung und der negative Druck viel größer waren. Auch da muss die Dichte am Ende genau die kritische gewesen sein (deswegen bemüht man ja die kosmische Inflation als Erklärung des Flachheitsproblems). D.h. die gravitative Abstoßung bzw. die Größe des negativen Drucks spielt letztlich keine Rolle für den Wert der kritischen Dichte oder die Raumkrümmung.

    Stimmt, am Ende der Inflation war die Dichte praktisch genau die kritische Dichte.

    Aber wie schließt du damit auf deinen letzten Satz?
    Die kritische Dichte ist zeitabhängig und definiert als
    https://upload.wikimedia.org/wikipedia/en/math/7/2/1/7215e34146072041c98e6a266ba1ee4e.png
    In den Hubble-Parameter fließt selbstverständlich die kosmologische Konstante ein?

  55. @Niels

    Danke für die Erläuterungen. Puh, da muss ich mich heute abend mal in Ruhe durchbeißen.

    Wie gesagt, eine Stärke des Inflationsmodells ist es, dass das gerade nicht so sein muss.

    Das extrem frühe Universum kann eine fast beliebige Krümmung und Dichte gehabt haben.

    Ich hab‘ mich aber auch missverständlich ausgedrückt. Mit „Anfangs, als das Universum noch klein war“ bezog ich mich auf die Zeit nach der Inflationsphase, während der die Materiedichte noch den Hubble-Parameter dominierte, und ausdrücklich nicht gemeint war die Inflationsphase, weil diese, wie Du sagst, eine beliebige Abweichung von der kritischen Dichte ausgebügelt hat. Letzteres wiederum meinte ich mit

    „Das hat aber auch schon während der kosmischen Inflationsphase gegolten, als die gravitative Abstoßung und der negative Druck viel größer waren. Auch da muss die Dichte am Ende genau die kritische gewesen sein (deswegen bemüht man ja die kosmische Inflation als Erklärung des Flachheitsproblems).“

    Nach der Inflation hätte eine kleine Abweichung von der kritischen Dichte dafür gesorgt, dass diese sehr schnell angewaschsen wäre: bei mehr als der kritischen Dichte hätte sich die Materie zusammengeballt und die Dichte wäre gestiegen; bei weniger als der kritischen Dichte wären die Abstände so schnell gewachsen, dass die Dichte schnell abgenommen hätte. Dass wir nach 13,7 Milliarden Jahren immer noch die kritische Dichte haben, ist eigentlich so unwahrscheinlich, wie ein auf der Spitze stehender Bleistift. Und das ist das Flachheitsproblem, dass die kosmische Inflation damit löst, die kritische Dichte ungeheuer genau eingestellt zu haben. Der Bleistift war perfekt senkrecht ausgerichtet.

    D.h. die gravitative Abstoßung bzw. die Größe des negativen Drucks spielt letztlich keine Rolle für den Wert der kritischen Dichte oder die Raumkrümmung.

    Stimmt, am Ende der Inflation war die Dichte praktisch genau die kritische Dichte.

    Aber wie schließt du damit auf deinen letzten Satz?

    SCHWAR_As Punkt war doch, dass die abstoßende Gravitation der Dunklen Energie die Raumkrümmung beeinflusst, und mein Argument oben sagt, dass das offenbar nicht so ist, weil selbst während der Inflation, als der negative Druck ungeheuer viel größer war als im heutigen Universum, die abstoßende Gravitation den Raum nicht anders krümmte als heute: kurz vor Ende der Inflation wie auch kurz danach war das Weltall flach, obwohl sich die abstoßende Kraft der Vakuumenergie um zig Zehnerpotenzen verringert hatte. Ich hoffe, das Argument ist korrekt?

  56. @Alderamin
    Oh, dann hab ich den ganzen Mist umsonst geschrieben?

    Die Dichte der Materie nahm mit zunehmender Ausdehnung ab, die kritische Dichte im gleichen Maße
    ist aber trotzdem falsch. 😉

    Nach der Inflation hätte eine kleine Abweichung von der kritischen Dichte dafür gesorgt, dass diese sehr schnell angewaschsen wäre: bei mehr als der kritischen Dichte hätte sich die Materie zusammengeballt und die Dichte wäre gestiegen; bei weniger als der kritischen Dichte wären die Abstände so schnell gewachsen, dass die Dichte schnell abgenommen hätte. Dass wir nach 13,7 Milliarden Jahren immer noch die kritische Dichte haben, ist eigentlich so unwahrscheinlich, wie ein auf der Spitze stehender Bleistift.

    Na ja, wie beschrieben:

    Jede Abweichung ist/wäre ungefähr um einen Faktor 10^60 angestiegen. (So stehts in der Wikipedia mit Quellenangabe. Ich weiß nicht mehr, wo ich meine 10^-70 herhabe.)
    Das bedeutet, dass für eine heutige, den Beobachtungen entsprechende maximale Abweichung von 0.05 die Abweichung zu Beginn der „normalen Expansion“ nicht größer gewesen sein darf als 5*10^-62. (5*10^-62 * 10^60 = 0.05)
    Das ist das Flachheitsproblem.
    Den extrem unwahrscheinlichen Ausgangszustand mit einer so kleinen Abweichungen von der Flachheit kann man schon mit einem auf der Spitze stehender Bleistift vergleichen.
    Mir persönlich hilft diese Veranschaulichung aber nicht wahnsinnig weiter…

    kurz vor Ende der Inflation wie auch kurz danach war das Weltall flach, obwohl sich die abstoßende Kraft der Vakuumenergie um zig Zehnerpotenzen verringert hatte

    Das liegt daran, dass sich der Dichteparameter des Inflationfeldes in den Dichteparameter für die Strahlung umgewandelt hat.
    Das Potential des zugehörigen Skalarfeldes wandelt sich zu massiven Teilchen um, die sofort zerfallen.
    Hier ist ein hübsches Bildchen für die Entwicklung der verschiedenen Dichteparameter mit der Zeit:
    https://ned.ipac.caltech.edu/level5/March03/Lineweaver/Figures/figure3.jpg

    SCHWAR_As Punkt war doch, dass die abstoßende Gravitation der Dunklen Energie die Raumkrümmung beeinflusst, und mein Argument oben sagt, dass das offenbar nicht so ist, weil selbst während der Inflation, als der negative Druck ungeheuer viel größer war als im heutigen Universum, die abstoßende Gravitation den Raum nicht anders krümmte als heute

    Wie oben geschrieben gilt:

    (1-Ω)*H²*a² = -k = konstant
    H²*a² ist nach Definition des Hubble-Parameters gleich der ersten Ableitung des Skalenfaktors

    H²*a² = a’², also

    (1-Ω)*a’² = konstant

    Die zeitliche Entwicklung der Geschwindigkeit der Expansion, also a‘, hängt aber doch durchaus von der Größe des negativen Drucks ab, oder?
    Weil die Gleichung konstant sein muss, hängt auch Ω davon ab. Ω wiederum hängt mit der Krümmung des Raumes zusammen.

    kurz vor Ende der Inflation wie auch kurz danach war das Weltall flach, obwohl sich die abstoßende Kraft der Vakuumenergie um zig Zehnerpotenzen verringert hatte.

    Ja, aber vor dem Ende der Inflationphase wurde die Größe der Abweichung von 1 kleiner gemacht, nach der Inflationphase wuchs die Größe der Abweichung von 1 dagegen sehr stark an.

    Das Universum wechselt hier sehr schnell von (extrem) beschleunigter Expansion zu stark abgebremster Expansion.

    (Das ist ja nichts Neues. Sehr, sehr viel später wechselt das Universum aufgrund der dunkle Energie dann wieder von dieser gebremsten Expansion zurück zu einer beschleunigten Expansion.)

  57. @Niels

    Sicher hast Du das nicht umsonst geschrieben, bisher habe ich die Zusammenhänge ja nur aus populärwissenschaftlichen Büchern (Guth, Greene, Krauss) ohne Formeln gelernt, wäre schön, wenn ich da mal zu einem tieferen Verständnis käme. Ich werde mir, wie gesagt, die Formeln heute abend mal in Ruhe ansehen, und ggf. rückfragen. Eine Rückfrage habe ich schon:

    Wenn es wirklich so klar wäre, dass unser Universum aus dem Nichts entstanden wäre und einen Null-Energie-Gehalt hätte, würde sich dieses Flachheitsproblem natürlich in Luft auflösen. Völlig ohne Inflation.
    Der Zahlenwert wäre einfach von Beginn an genau Ω=1 und fertig.

    Es braucht die Inflation aber doch auch noch in diesem Fall wegen des Homogenitätsproblems, also um zu erklären, warum weit getrennte Orte in der kosmischen Hintergrundstrahlung fast identische Temperaturen haben, und um zu erklären, wie winzige Inhomogenitäten aus Quantenfluktuationen entstanden, nicht wahr?

  58. @Alderamin
    Richtig.
    Ersteres nennt man aber üblicherweise das Horizontproblem.

    Häufig wird auch noch genannt, dass die Inflation das sogenannte Monopol-Problem löst.
    Viele Ansätze für Große vereinheitlichte Theorien sagen voraus, dass im extrem frühen Universum sehr viele magnetische Monopole oder andere seltsame Objekte entstanden sein müssen.
    Magnetische Monopole haben/hätten eine Masse von etwa 10^15 GeV (!) und tragen eine magnetische Ladung.
    Allerdings können wir keine solchen Objekte beobachten. Wenn es solche Objekte gibt, dann müssen es extrem wenige sein und sie müssen außerdem einen sehr großen Abstand voneinander haben.
    Die Theorien sagen allerdings die Existenz sehr vieler solcher magnetischer Monopole im frühen Kosmos voraus. In einem normal expandierenden Universum müssten wir andauernd auf sie stoßen.

    Die Inflation löst auch dieses Problem:
    Während der Erzeugungsphase magnetischer Monopole oder danach blähte die Inflationsepoche die Raumzeit so stark auf, dass die Monopoldichte extrem ausgedünnt wurde
    In unserem beobachtbaren Universum blieben dadurch keine oder praktisch keine übrig.

    Die Inflation löst also das Horizontproblem, das Flachheitsproblem, das Monopolproblem und das Problem der Dichtefluktuationen.
    Alle diese Dinge lassen sich aber auch durch verschiedene andere Annahmen, beispielsweise aus der Stringtheorie, auflösen.
    Das Inflationsmodell hat dabei den großen Vorteil, dass es alle diese Probleme auf einmal lösen kann.
    Andere bisher gefundene Lösungen klären immer nur ein einziges dieser Probleme.

    Den von dir zitierten Abschnitt habe ich geschrieben, weil ich darauf hinweisen wollte, dass die Inflation als Lösung des Flachheitsproblems schon ne ganze Weile Standardwissen ist, das man z.B. in jedem nicht völlig veralteten Lehrbuch über Kosmologie findet.
    Das zero-energy-universe-Hypothese ist noch lange kein Konsens.
    Da ersetzt man ja eigentlich auch nur das Problem des finetuning der kritischen Dichte durch das finetuning des Gesamtenergie-Gehaltes.
    Und das man dann trotzdem noch Inflation benötigt ist doch eine Schwäche der Hypothese, kein Vorteil.

    Wobei natürlich auch die Inflation ein gewisses finetuning benötigt. (graceful exit
    problem, …)
    Um das zu lösen, braucht man dann die sogenannte ewige Inflation (eternal chaotic inflation).
    Wobei man sich dann zwangsläufig das ganze Zeug mit dem Multiversum und der unendlichen Zahl inflationärer Universen einhandelt.
    Das kommt mir auch nicht besonders befriedigend vor. Mit Unendlichkeit kann man schließlich alles totschlagen.

  59. @Niels

    Nochmal danke für die Formeln oben, ich komme bei den Umformungen zwar nicht ganz mit, aber die Schlussfolgerung in Bezug auf die Inflation ist klar.

    Da ersetzt man ja eigentlich auch nur das Problem des finetuning der kritischen Dichte durch das finetuning des Gesamtenergie-Gehaltes.
    Und das man dann trotzdem noch Inflation benötigt ist doch eine Schwäche der Hypothese, kein Vorteil.

    Das sehe ich nicht so, denn wenn man ein Universum mit irgendeinem Wert an Energie anfangen muss, dann ist 0 irgendwie der plausibelste. Bei jedem bestimmten Wert fragt man sich, wieso gerade dieser Wert? Bei 0 fängt man mit Nichts an, und Nichts ist ein plausibler Anfangspunkt. Du meinst, die Nullenergie sei eine Zusatzannahme, ich würde eher sagen, jede andere Energie als 0 wäre eine Zusatzannahme.

    Um das zu lösen, braucht man dann die sogenannte ewige Inflation (eternal chaotic inflation).
    Wobei man sich dann zwangsläufig das ganze Zeug mit dem Multiversum und der unendlichen Zahl inflationärer Universen einhandelt.
    Das kommt mir auch nicht besonders befriedigend vor. Mit Unendlichkeit kann man schließlich alles totschlagen.

    Die ewige Inflation ist aber eine wunderbare Erklärung dafür, warum unser Universum so schön feinabgestimmt ist, dass es uns hervorbringen konnte. Jede Raumblase, die in der um uns herum ewig weiterlaufenden Inflation entsteht und ein Universum gebiert, hat möglicherweise ihre komplett eigenen Naturgesetze und in fast allen Fällen entsteht dabei nur irgendein homogener Energiebrei. Aber wegen der großen Anzahl von „Versuchen“ muss auch hin und wieder mal ein Universum mit Materie und mit mit Naturgesetzen herauskommen, die Chemie und Biologie möglich machen. Ohne ein Multiversum hast Du ein großes Problem zu erklären, warum es gleich beim ersten Mal genau gepasst hat. Dann müsstest Du in irgendeiner GUT einen Grund dafür finden, warum die physikalischen Naturkonstanten genau so sein müssen wie sie es sind.

    Die Multiversums-Hypothese erklärt mir plausibel, warum unsere Existenz letztlich unvermeidlich war, und das ist ihre eigentliche Stärke.

    Siehe auch das Interview mit Krauss hier.

  60. @Alderamin

    Nochmal danke für die Formeln oben, ich komme bei den Umformungen zwar nicht ganz mit

    Liegt wahrscheinlich daran, dass ich mich verschrieben habe.

    Es muss statt

    [(1/Ω)-1]*ρ*a² = konstant vielmehr

    [{(1/Ω)^-1}-1]*ρ*a² = konstant
    heißen.
    Bei den danach kommenden Formeln muss man dann auch immer [(1/Ω)-1] durch [{(1/Ω)^-1}-1] ersetzen.

    Das bedeutet, dass der Dichteparameter Ω für den Zeitpunkt mit der Rotverscheibung z einen Wert
    größer als 1/[1+0,05/(1+z)] und
    kleiner als 1/[1+0,05/(1+z)] haben musste.

    Das hat gestimmt. Darauf mit der falschen Formel zu kommen ist aber natürlich ziemlich schwierig. 😉

    Zur Herleitung allgemein:
    Die Friedmanngleichung ist
    https://upload.wikimedia.org/wikipedia/de/math/6/b/0/6b0d18e2e06e20378511db06a5102be6.png
    (Wobei wir Theoretiker sind und deswegen c=1 setzen.)

    Die linke Seite ist gleich H².
    Jetzt teilt man auf beiden Seiten durch H² und beachtet, dass die kritische Dichte folgendermaßen definiert ist:
    https://upload.wikimedia.org/wikipedia/en/math/7/2/1/7215e34146072041c98e6a266ba1ee4e.png
    (Diagramme für den Verlauf des Hubble-Parameters für unser Universum kennst du, oder? Das Ganze einfach nur quadrieren, dann hast du den Verlauf der kritischen Dichte.
    Wenn du möchtest, kann auch mal probieren, ob ich es hinkriege, das bei Wolfram-Alpha zu plotten.)

    Als Ergebnis bekommt man
    1 = ρ/ρc – k/(H²*a²) + lamdba/(3*H²)
    Frü den Dichteparameter Ωρ gilt Ωρ = ρ/ρc, der Dichteparamter der dunklen Energie Ωlambda ist Ωlambda = lambda/(3*H²) und für den Dichteparameter des Universums Ω gilt Ω = (ρ/ρc) + Ωlambda.
    Man bekommt also
    1 = Ω – k/(H²*a²) und daraus ganz leicht (1-Ω)*H²*a² = -k = konstant.

    Um auf
    [{(1/Ω)^-1}-1]*ρ*a² = konstant
    zu kommen, schauen wir uns wieder die Friedmann-Gleichung von oben an.
    Weil wir uns nur für die strahlungs- und materiedominierten Epochen interessieren, lassen wir als Näherung den lambda-Term weg.

    Dann multiplizieren wir auf beiden Seiten mit 3/(8*Pi*G*ρ).
    Mit Ω = ρ/ρc (der Dunkle-Energie-Term fällt auch hier weg) kommen wir auf die obige Gleichung.

    Das machen wir übrigens nur, damit wir zur Veranschaulichung konkrete Zahlenwerte berechnen können.
    Für Aussagen über die generelle Entwicklung von Ω reicht die Gleichung (1-Ω)**a² = konstant eigentlich schon aus.
    Wir wissen, dass H²*a² = a’² ist und wir wissen, dass a ~ t^(1/2) für die strahlungsdominierte Ära und a ~ t^(2/3) für die materiedominierte Ära.
    Dann kann man a‘ und a’² ausrechnen und allgemeine Aussagen treffen, wie sich Ω verhalten muss, damit die Gleichung konstant bleibt.

    Zahlenwerte sind aber natürlich immer ne tolle Sache und deswegen betrachten wir
    [{(1/Ω)^-1}-1]*ρ*a² = konstant.
    Dabei gibt es natürlich ein paar Fehlerquellen.
    Zum einen darf man natürlich nicht einfach lambda = 0 setzen, auch nicht fürs frühe Universum.
    Zum anderen setzen wir auch noch Ω = 1 plusminus 0,05 für unser heutiges Universum ein, wissen, dass davon ein Anteil von 70% von der dunklen Energie stammt und tun dann im Anschluss trotzdem so, als wären wir noch in der völlig materiedominierten Epoche und könnten die dunkle Energie vernachlässigen.
    Unsere Ergebnisse sind also vermutlich sehr ungenau.

    Für Zahlenwerte wie 10^-60 ist es aber völlig egal, ob wir uns um ein paar Größenordnungen verschätzt haben oder nicht.

  61. (Neuer Beitrag, weil der Spam-Filter mich bei mehr als zwei Links rausschmeißt.)

    Das sehe ich nicht so, denn wenn man ein Universum mit irgendeinem Wert an Energie anfangen muss, dann ist 0 irgendwie der plausibelste. Bei jedem bestimmten Wert fragt man sich, wieso gerade dieser Wert? Bei 0 fängt man mit Nichts an, und Nichts ist ein plausibler Anfangspunkt.

    Ich hab noch kein einziges der neueren Paper über das zero-energy-universe gelesen, die die Idee seit Kurzem wieder besonders populär gemacht haben.

    Deswegen kann ich nicht beurteilen, welche neuen Argumente da auftauchen und wie überzeugend diese sind.

    Die Grundidee selbst ist allerdings schon ziemlich alt, sogar älter als die Inflation.
    https://pdfcast.org/pdf/tryton1973-vac-fluc
    Das Problem bei diesem Ur-Modell ist, dass man eine Art „Hintergrundraum“ braucht.
    So etwas empfanden viele Forscher als unelegant.

    Das wurde 1982 von Vilenkin gelöst. Dort wird so etwas nicht mehr gebraucht.
    https://mukto-mona.net/science/physics/a_vilinkin/universe_from_nothing.pdf

    Soviel ich weiß bezieht sich Guth in „The Inflationary Universe“ auf das Vilenkin-Modell.
    (Das ist das Buch von Guth, dass du dazu gelesen hast, richtig?)

    Dieses Modell hat aber ebenfalls philosophische Probleme:
    Der Tunneleffekt, durch den das Universum hier aus dem „Nichts“ entstand, wird durch die Gesetze der Quantenphysik beschrieben.
    Das „Nichts“ muss also diesen Gesetzen gehorchen, die Naturgesetze müssen daher schon existiert haben, bevor es überhaupt so etwas wie ein Universum gab.
    Insgesamt muss dieses „Nichts“ also auf eine ganz bestimmte Weise (aus unendlich vielen Möglichkeiten heraus) mathematisch wohldefiniertheit sein.
    Auch ein bisschen seltsam, oder?
    Ist dieses „Nichts“ dann überhaupt noch ein Nichts, oder nicht doch schon ein ganz bestimmter, perfekt feinabgestimmter Zustand?

    Man könnte mit einiger Berechtigung fragen, ob man das Problem so nicht nur um eine Ebene verschiebt, es aber dadurch nicht gelöst wird.

    Man kann sich außerdem streiten, ob ein solches „Nichts“ ein wirklich so viel plausiblerer Ausgangszustand ist als irgendein anderer Zustand, der dem Universum eben eine Energie ungleich Null mitgegeben würde.

    Wie gesagt, dieses Modell hat sich auch (noch?) nicht als Konsens durchgesetzt.

    Wobei man sich allgemein die Frage gefallen lassen muss, ob solche höchstwahrscheinlich niemals zu falsifizierenden Ideen überhaupt noch etwas mit harter Naturwissenschaft zu tun haben.

    Die ewige Inflation ist aber eine wunderbare Erklärung dafür, warum unser Universum so schön feinabgestimmt ist, dass es uns hervorbringen konnte.
    […]
    wegen der großen Anzahl von „Versuchen“ muss auch hin und wieder mal ein Universum mit Materie und mit mit Naturgesetzen herauskommen

    Ist mir bekannt.
    Wie gesagt, mit Unendlichkeit kann man immer alles totschlagen. 😉

    Ohne ein Multiversum hast Du ein großes Problem zu erklären, warum es gleich beim ersten Mal genau gepasst hat. Dann müsstest Du in irgendeiner GUT einen Grund dafür finden, warum die physikalischen Naturkonstanten genau so sein müssen wie sie es sind.

    Würfelglück?

    Wenn man einen Würfel mit 10^100 Seiten wirft, liegt am Ende trotzdem immer nur eine Seite oben.
    Natürlich ist es extrem unwahrscheinlich, dass gerade diese Seite oben liegt. Aber irgendeine Seite muss nun mal oben liegen. Jede andere Seite wäre dann ganz genau so unwahrscheinlich.
    Wenn die falsche Seite oben läge, würden sich vielleicht völlig anders strukturierte Lebensformen dieselbe Frage nach einer völlig anderen Feinabstimmung stellen.

    Okay, solche Argumente sind natürlich auch nicht besonders befriedigend.
    So wie allgemein das anthropische Prinzip ein bisschen unschön ist.

    Die Multiversums-Hypothese erklärt mir plausibel, warum unsere Existenz letztlich unvermeidlich war, und das ist ihre eigentliche Stärke.

    Auch klar.
    Ein Multiversum mit unendlich vielen Universen verursacht mir aber einfach irgendwie Bauchgrimmen.
    Man kann sich auch fragen, ob diese Lösung nicht Ockhams Rasiermesser widerspricht.

    Aufgrund desselben Bauchgrimmens finde ich z.B. auch die Viele-Welten-Interpretation nicht besonders überzeugend.
    Das geht ja ziemlich vielen anderen Physikern auch nicht anders.
    (Wobei andere diese Interpretation genau deswegen so überzeugend finden.)

    Kann aber schon sein, dass meine Position eine veraltetet Position ist, die langsam aussterben wird.
    Wäre natürlich schade, wenn ich in meinem Alter schon zum alten Eisen gehören würde. 😉

  62. @Niels

    Soviel ich weiß bezieht sich Guth in „The Inflationary Universe“ auf das Vilenkin-Modell.
    (Das ist das Buch von Guth, dass du dazu gelesen hast, richtig?)

    Genau, hab‘ ich im englischen Original. Da braucht es aber ein paar Gramm Ausgangsmaterial zu einem falschen Vakuum komprimierter Materie, um die Inflation anzustoßen.

    Dieses Modell hat aber ebenfalls philosophische Probleme:
    Der Tunneleffekt, durch den das Universum hier aus dem „Nichts“ entstand, wird durch die Gesetze der Quantenphysik beschrieben.
    Das „Nichts“ muss also diesen Gesetzen gehorchen, die Naturgesetze müssen daher schon existiert haben, bevor es überhaupt so etwas wie ein Universum gab.
    Insgesamt muss dieses „Nichts“ also auf eine ganz bestimmte Weise (aus unendlich vielen Möglichkeiten heraus) mathematisch wohldefiniertheit sein.
    Auch ein bisschen seltsam, oder?
    Ist dieses „Nichts“ dann überhaupt noch ein Nichts, oder nicht doch schon ein ganz bestimmter, perfekt feinabgestimmter Zustand?

    Man könnte mit einiger Berechtigung fragen, ob man das Problem so nicht nur um eine Ebene verschiebt, es aber dadurch nicht gelöst wird.

    Man kann sich außerdem streiten, ob ein solches „Nichts“ ein wirklich so viel plausiblerer Ausgangszustand ist als irgendein anderer Zustand, der dem Universum eben eine Energie ungleich Null mitgegeben würde.

    Lies mal das oben von mir verlinkte Interview mit Krauss durch, und am besten anschließend sein Buch zum Thema. Da setzt er sich explizit mit diesem Vorwurf auseinander. Er meint wirklich „Nichts“. Kein Raum,, keine Zeit, keine Naturgesetze. Das einzige, was es braucht, ist Krauss gemäß eine Unschärferelation, die aus Nichts einen Raum mit zufälligen Naturgesetzen gebiert. Ein unendlich kleiner Raum oder unendlich kurze Zeit ist durch die Unschärferelation einfach unmöglich. „Nichts“ ist instabil.

    Die Heisenbergsche Unschärferelation als Urmutter des Universums und aller Naturgesetze. Das hat was.

    Würfelglück?
    Wenn man einen Würfel mit 10^100 Seiten wirft, liegt am Ende trotzdem immer nur eine Seite oben.

    Das anthropische Prinzip funktioniert nur, wenn man einen genügend großen Wahrscheinlichkeitsraum hat, z.B. viele Planeten, von denen dann nur wenige Leben hervorbringen und mindestens einer davon intelligentes. Das Würfelglück stellt nur sich dann mit plausibler Wahrscheinlichkeit irgendwo ein, wenn genug Würfel geworfen werden. Ein Multiversum leistet das, ein einzelnes Universum wie unseres wäre ein Lotto-Hauptgewinn im ersten Versuch, das würde mir Bauchschmerzen verursachen und denn Kreationisten in die Hände spielen.

    Kann aber schon sein, dass meine Position eine veraltetet Position ist, die langsam aussterben wird.
    Wäre natürlich schade, wenn ich in meinem Alter schon zum alten Eisen gehören würde. 😉

    Och, Du bist bestimmt jünger als ich, ich gehe mit großen Schritten auf die 50 zu. Man lernt bis zum Lebensende, auch wenn’s zunehmend schwieriger wird. 😉

  63. @Alderamin

    Niels schrieb:

    Soviel ich weiß bezieht sich Guth in „The Inflationary Universe“ auf das Vilenkin-Modell.

    Alderamin schrieb:
    Genau, hab‘ ich im englischen Original. Da braucht es aber ein paar Gramm Ausgangsmaterial

    Ich glaub ich versteh nicht, was du meinst?

    Beim Vilenkin-Modell braucht man ausdrücklich kein Ausgangsmaterial.

    Das ist doch genau das „quantum tunneling from nothing“-Modell.

    Guth weiß das doch ebenfalls.
    The Inflationary Universe, Seite 275:

    Putting (Anm.: ART und Quantenphysik) together, one can imagine that the universe started in the total empty geometry – absolute nothingness – and then made a quantum tunneling transition to a nonempty state. Calculations show that a universe created this way would typically be subatomic in size, but that is no problem
    […]
    Vilenkin was able to invoke inflation to enlarge the universe to its current size.

    Wenn man ein paar Gramm Ausgangsmaterial braucht bespricht Guth also gerade ein anderes Modell.

    Alderamin schrieb:
    Lies mal das oben von mir verlinkte Interview mit Krauss durch, und am besten anschließend sein Buch zum Thema. Da setzt er sich explizit mit diesem Vorwurf auseinander. Er meint wirklich „Nichts“. Kein Raum,, keine Zeit, keine Naturgesetze.

    Wenn er das wirklich so meint, versteht er das Vilenkin-Modell aber anders als ich und zufällig auch anders als Vilenkin selbst.
    Kein Raum und keine Zeit ist richtig, keine Naturgesetze eher nicht.

    Vilenkin hat auch ein Buch geschrieben, in dem er seine Erfindung, „Universum aus dem Nichts“, selbst vorstellt.
    Alex Vilenkin : Many Worlds in One: The Search for Other Universes
    Dort kommt allerdings kein einziges Mal das Wort Gott vor und es hat auch kein Nachwort von Richard Dawkins. Vermutlich deswegen ist es im Vergleich mit Krauss Buch völlig unbekannt.

    Vilenkin schreibt (Seite 180) über das Tryon-Modell mit dem „Hintergrundraum“:

    A more fundamental problem is that Tryon’s scenario does not really explain the origin of the universe. A quantum fluctuation of the vacuum assumes that there was a vacuum of some pre-existing space. And we now know that the „vacuum“ is very different from „nothing“. Vacuum, or empty space, has energy and tension, it can bend a warp, so it is unquestionably something.
    As Alan Guth wrote, „In this context, a proposal that the universe was created from empty space is no more fundamental than a proposal that the universe was spawned by a piece of rubber. It might be true, but one would still want to ask where the piece of rubber came from.“

    Auf Seite 185 schreibt er dann über sein eigenes Modell, das dieses „fundamental problem“ eben nicht aufweist:

    The concept of a universe materializing out of nothing boggles the mind. What exactly is meant by „nothing“? If this „nothing“ could tunnel into something, what could have caused the primary tunneling event? And what about energy conservaton?
    The initial state prior to the tunneling is a universe of vanishing radius, that is, no universe at all. There is no matter and no space in this very peculiar state. Also, there is no time. Time has meaning only if something is happening in the universe. We measure time using periodic processes, like the rotation of the Earth about its axis, or its motion around the Sun. In the absence of space and matter, time is impossible to define.
    […]
    And yet the state of „nothing“ cannot be identified with absolute nothingness. The tunneling is described by the laws of quantum mechanics, and thus „nothing“ should be subjected to these laws.
    The laws must have existed, even though there was no universe.

    Alderamin schrieb:
    Ein Multiversum leistet das, ein einzelnes Universum wie unseres wäre ein Lotto-Hauptgewinn im ersten Versuch, das würde mir Bauchschmerzen verursachen

    Mir auch. Multiversen finde ich aber sogar noch schlimmer.

    Andererseits sind solche Sachen in der Physik meiner Erfahrung nach auch manchmal einfach Gewohnheitssache. In ein paar Jahren finde ich dieses Konzept vielleicht ganz normal, mal sehen. 😉

    Aber um mal wieder zu unserem Ausgangspunkt, dem Flachheitsproblem, zurückzukommen:

    Das Problem mit der Idee einer Gesantebergie von Null ist gleich zu Beginn, dass es gar nicht klar, was mit dem Ausdruck „Gesamtenergie des Universums“ überhaupt gemeint ist.
    Ob und wie man diesen Begriff definieren könnte ist ein aktuelles, nicht geklärtes Forschungsproblem.
    Ich verlinke hier einfach mal eine Zusammenfassung, da ich das selbst auch nicht besser ausdrücken könnte:
    https://physicsforums.com/showthread.php?t=506985

    Wie dort aufgeführt erhält man z.B. durch die Wahl eines ganz bestimmten Pseudo-Tensors als erhaltene „Gesamtenergie“ das Ergebnis, dass diese so definierte „Gesamtenergie“ für jedes geschlossene Universum null sein muss.
    (Zumindest in kartesischen Koordinaten. In Kugelkoordinaten ist die Energie unendlich.
    Deswegen sind Pseudo-Tensoren doof.)
    Eine erhaltene „Gesamtenergie“ mit dem Wert Null nützt uns für die Lösung des Flachheitsproblem also in unter Umständen überhaupt Fall gar nichts.
    Die spezielle „Gesamtenergie„ nach Berman erlaubt jedenfalls, dass beliebige geschlossene Universen aus dem „Nichts“ entstehen.

    Die Sache mit dem zero-energy-universe beruht also erst mal auf dem Energiekonzept der Quantenphysik und der Behauptung, dass es zu diesem quantenmechanischen Energiebegriff auch ein passendes erhaltenes Analogon für die ART bzw. für eine noch zu findende Quantengravitation gibt.
    Das ist wie gesagt nur eine unbewiesene Behauptung, also ebenfalls eine Hypothese. Niemand weiß, ob das überhaupt möglich ist.
    Bis jetzt hat man jedenfalls auch nach Jahrzenten intensiver Forschung so eine erhaltene Gesamtenergie nicht gefunden.
    Man konnte aus den Gleichungen bisher noch nicht einmal ableiten, ob so etwas überhaupt existiert oder nicht.

    Für eine abschließende Beurteilung des „quantum tunneling from nothing“-Modells brauchen wir höchstwahrscheinlich also mal wieder eine funktionierende Quantengravitation.

    Ob die Entstehung des Universums aus dem Nichts bzw. aus einer Quantenfluktuation heraus uns überhaupt beim Flachheitsproblem weiterhilft, weiß man also überhaupt gar nicht.
    Das muss man als Zusatzannahme fordern.

    Aber selbst wenn das alles nicht so wäre, wir eine passende „Gesamtenergie“ definieren könnten und das „quantum tunneling from nothing“-Modell das Flachheitsproblem lösen würde:
    Dass das Universum als Quantenfluktuation begann ist ja erst einmal nur eine von dutzenden bekannten Möglichkeiten und keine Zwangsläufigkeit.

    Das Problem der ersten Ursache löst man z.B. auch dadurch, dass man irgendetwas Ewiges postuliert.

    Andere Modelle für die Ursache des Urknalles umfassen etwa

    • die Mathematical universe hypothesis
    • ewige Branen-Welten
    • das ewige Hartle-Hawking Universe
    • das Cosmological natural selection – Modell
    • ein ewiges „Hintergrunduniversum“ (aus dem heraus andere Universen entstehen, z.B. durch Quantenfluktuationen)
    • verschiedene Arten von ekpyrotischen/zyklischen Universen.

    Gerade diese letzte Idee eines zyklischen Universums ist momentan besonders beliebt und es wurden sehr viele verschiedene Arten eines solchen Universums entwickelt.
    Etwa

    • das Baum–Frampton Cyclic Model
    • die Conformal cyclic cosmology
    • verschiedene Modelle aus der M-Theorie/Branen-Kosmologie, bei denen M-Branen zusammenstoßen und so den Urknall verursachen
    • verschiedene Modelle aus der Schleifenquantengravitation/Loop quantum cosmology.

    (Was sich hinter diesen ganzen genannten Begriffen verstreckt, musst du einfach mal googeln.
    Da kenn ich mich erstens nicht besonders gut aus und zweitens bin ich zu faul, das jeweils auch noch auszuführen.
    Ich hab extra die bekanntesten englischen Bezeichnungen gewählt. Damit müsste leicht etwas zu finden sein.)

    Mit Sicherheit gibt’s noch ne Menge weiterer Vorschläge für Ursachen des Urknalls, von denen ich einfach noch nie gehört habe.

    Es gibt momentan keinen überzeugenden Grund, warum man das „quantum tunneling from nothing“-Modell Vilenkins gegenüber den anderen Modellen bevorzugen sollte.
    Na gut, man kann die „Entstehung aus dem Nichts“ als besonders elegant empfinden. Aber ein echtes Argument ist das eigentlich nicht.

    Soweit ich das überblicke, wurde beim Vergleich einiger Modelle sogar gezeigt, dass durch Messungen im beobachtbaren Universums prinzipiell nicht zwischen ihnen unterschieden werden kann.

    Ui, das war jetzt aber viel zu lang. Sorry.

  64. @Niels

    Alderamin schrieb:
    Genau, hab‘ ich im englischen Original. Da braucht es aber ein paar Gramm Ausgangsmaterial

    Ich glaub ich versteh nicht, was du meinst?

    Beim Vilenkin-Modell braucht man ausdrücklich kein Ausgangsmaterial.

    Ich zitiere mal aus dem Inflationary Universe, S. 254 f:

    The Cosmic Cookbook entry for an inflationary universe, on the other hand, looks as simple as a meringue. In this case the natural starting time would be the onset of inflation. In contrast to the standard big bang recipe, the inflationary version calls for only a single ingredient: a region of false vacuum 10^-26 centimeters across is all the recipe demands. While the mass required for the previous recipe [Anm.: Standard Big Bang, nach der Inflation] was 10^32 solar masses, the mass in this case is only 10^-32 solar masses. The sign of an exponent can make a big difference: in more easily recognizable units, the required mass is about 25 grams, or roughly one ounce! So, in the inflation theory the universe evolves from essentially nothing at all, which is why I frequently refer to it as the ultimate free lunch.

    Daher hatte ich das. Aber Du hast natürlich recht. Es ist schon ein paar Jahre her, dass ich das Buch gelesen habe, die Vilenkin-Passage war nicht hängen geblieben, die Passage oben schon.

    Dort kommt allerdings kein einziges Mal das Wort Gott vor und es hat auch kein Nachwort von Richard Dawkins. Vermutlich deswegen ist es im Vergleich mit Krauss Buch völlig unbekannt.

    Hmm, magst Du Krauss irgendwie nicht? 😉 Er hat auch schon vorher bekannte Bücher geschrieben wie „The Physics of Star Trek“, in denen Dawkins kein Nachwort geschrieben hat…

    Alderamin schrieb:
    Lies mal das oben von mir verlinkte Interview mit Krauss durch, und am besten anschließend sein Buch zum Thema. Da setzt er sich explizit mit diesem Vorwurf auseinander. Er meint wirklich „Nichts“. Kein Raum, keine Zeit, keine Naturgesetze.

    Wenn er das wirklich so meint, versteht er das Vilenkin-Modell aber anders als ich und zufällig auch anders als Vilenkin selbst.

    Auf S. 273 im Inflationary Universe schreibt Guth:

    In 1982, Alexander Vilenkin of Tufts University proposed an extension of Tryon’s idea. He suggested that the universe was created by quantum processes starting from „literally nothing“, meaning not only the absence of matter, but the absence of space and time as well.

    Es fragt sich dann allerdings, welche Naturgesetze dann noch übrig bleiben sollen, wenn man keine Materie, keinen Raum und keine Zeit mehr hat. Das Unschärfeprinzip greift dann sozusagen an dem Wörtchen „keine“ an und hat somit einen Hebel, um etwas zu bewirken, aber alle anderen Naturgesetze sind ansonsten belanglos. Dann kann man doch auch sagen, die Naturgesetze seien erst mit Raum und Zeit entstanden, oder?

    Aber was soll ich Krauss oder das Nullenergie-Universum verteidigen, ich hab‘ halt sein Buch gelesen, seine Argumentation schien mir einleuchtend, und er hat definitiv mehr Ahnung von dem Thema als ich, also werde ich ihm weder widersprechen, noch ihn in Schutz nehmen können. Ich nehme aus Deinem Post mit, dass es noch zahlreiche andere Möglichkeiten für den Beginn des Universums gegeben haben könnte (wobei, ohne das Buch gelesen zu haben, sondern nur einen Spiegel-Artikel darüber, sich wohl auch Hawking in seinem letzten Buch auf das Nullenergie-Universum bezogen haben soll, das sind dann schon zwei, oder mit Vilenkin drei). Es hat für mich einfach eine gewisse Eleganz, dass aus Nichts ein Universum geworden sein könnte. Oder viele.

  65. @Alderamin

    sich wohl auch Hawking in seinem letzten Buch auf das Nullenergie-Universum bezogen haben soll, das sind dann schon zwei, oder mit Vilenkin drei

    Klar. Da findet man auch noch sehr viele andere Forscher.
    Dieses Modell ist allgemein als Möglichkeit anerkennt.

    Hmm, magst Du Krauss irgendwie nicht? 😉

    Nö, das hast du falsch verstanden.
    Ich mag die Vermarktungsstrategie für sein neuen Buches nicht. Man sollte Religion völlig aus der Kosmologie raushalten. Das nervt mich ähnlich wie die Bezeichnung „Gottes“-Teilchen und ist genau so „hilfreich“ fürs Verständnis. Deswegen hat ein Nachwort von Richard Dawkins meiner Ansicht nach nichts in einem (poplär)wissenschaftlichen Buch zu suchen.
    Außerdem finde ich es schade, dass dieses Marketing-Konzept so durchschlagenden Erfolg hatte und die Idee der „Entstehung aus dem Nichts“ jetzt häufig Krauss zugesprochen wird.
    Während Vilenkins Buch leider extrem unbekannt ist.

    Auf diese Marketing-Strategie hatte Krauss selbst aber bestimmt extrem wenig Einfluss.
    Außerdem kann man natürlich argumentieren, dass es prinzipiell immer positiv ist, wenn sich Menschen mit Wissenschaft beschäftigen. Egal, wie sie dazu gebracht werden.

    Das Unschärfeprinzip greift dann sozusagen an dem Wörtchen „keine“ an und hat somit einen Hebel, um etwas zu bewirken, aber alle anderen Naturgesetze sind ansonsten belanglos. Dann kann man doch auch sagen, die Naturgesetze seien erst mit Raum und Zeit entstanden, oder

    Na ja, ich verstehe dieses Modell nach Lesen des zugehörigen Papers (hab ich oben verlinkt) einfach ein bisschen anders, als dass nur ein Unschärfeprinzip „im „Nichts““ nötig gewesen wäre.
    Vilenkin selbst meiner Meinung nach auch. Siehe Zitat.
    Aber da kenne ich mich wirklich nicht gut genug aus, da kann ich mich sehr leicht irren.

    Wie dieses „Nichts“ genau zu verstehen ist, spielt für unsere Diskussion über das flache Universum bzw. das zero-energy-universe aber eigentlich überhaupt keine Rolle.

    Aber was soll ich Krauss oder das Nullenergie-Universum verteidigen, ich hab‘ halt sein Buch gelesen, seine Argumentation schien mir einleuchtend
    […]
    Ich nehme aus Deinem Post mit, dass es noch zahlreiche andere Möglichkeiten für den Beginn des Universums gegeben haben könnte

    Du brauchst da nichts verteidigen. Ich erkenne das Modell doch an.
    Aber wie du selbst sagst, dass „quantum tunneling from nothing“-Modell ist eben nur ein Modell unter vielen.

    Es muss nicht unbedingt so gewesen sein.

    Du hättest auch noch mitnehmen sollen, dass es im Moment überhaupt keine anerkannte Definition für die „Gesamtenergie“ eines Universums gibt. Erst recht keine Definition einer „Gesamtenergie“, die dann mit der Expansion auch erhalten bleibt.
    Ob man so eine Definition finden kann und ob das dann überhaupt irgendwie mit der Energieerhaltung der QM zusammenpasst ist völlig unklar.
    Das geht zwar völlig gegen jede Intuition, aber bei der ART muss man sich nun mal ebenfalls von einigen geliebten Vorstellungen verabschieden, genau wie es auch bei der QM ist.

    Man redet beim zero-energy-universe also über ungelegte Eier. (Wobei man genau gesagt nicht einmal weiß, ob es überhaupt Hühner gibt.)

    Deswegen ist es keineswegs klar, ob eine „quantum tunneling from nothing“-Modell nach Vilenkin zwingend zu einem flachen Universum führen muss.
    Wie gesagt, beispielsweise sind bei der Wahl der „Gesamtenergie“ nach Berman auch alle geschlossenen Universen zero-energy-universes.
    Das Flachheitsproblem bleibt also erst einmal bestehen. Auch dann, wenn man vom Vilenkin-Modell ausgeht.

    Es hat für mich einfach eine gewisse Eleganz, dass aus Nichts ein Universum geworden sein könnte. Oder viele.

    Klar.
    Siehe oben:

    Na gut, man kann die „Entstehung aus dem Nichts“ als besonders elegant empfinden. Aber ein echtes Argument ist das eigentlich nicht.

    Ich finde die Idee auch sehr reizvoll. Das ist hier aber so änhlich wie bei den Interpretationen der QM. Mann kann ein Lieblingsmodell haben oder bestimmte Modelle ablehnen. Aber wenn man ehrlich ist, muss man zugeben, dass das einfach Geschmackssache ist. Unser momentanes Verständnis der Physik kann uns nur sagen, dass alle diese Möglichkeiten zutreffend sein könnten.

    Viele String-Theoretiker empfinden beispielsweise die String-Theorie als besonders elegant, während andere Quantengraviation-Theoretiker das ganz anders sehen.
    Eleganz ist immer etwas sehr Subjektives.

    Ich könnte mir auch vorstellen, dass da der kulturelle Hintergrund eine nicht ganz unbedeutende Rolle spielt.
    Für Mitglieder der westlichen Kulturen ist eine Entstehung aus dem Nichts vermutlich besonders plausibel. Für Inder, buddhistisch geprägte Kulturen, … hat vielleicht eher die Idee eines Zyklus einen besonders großen Reiz.
    (Wobei ich mich bei östlichen Kulturen und deren religiösen/kulturellen Auffassungen aber besser nicht zu weit aus dem Fenster lehne, da hab ich nämlich nur sehr wenig Ahnung.)

    Vielleicht hat diese „Eleganz“ also nur mit Gewohnheit zu tun und damit, welche Vorstellungen kulturell besonders tief verwurzelt und damit vertraut sind.

  66. @Niels

    Man sollte Religion völlig aus der Kosmologie raushalten. Das nervt mich ähnlich wie die Bezeichnung „Gottes“-Teilchen und ist genau so „hilfreich“ fürs Verständnis

    Na ja, aber die Frage nach dem Ursprung ist doch eines der zentralen Argumente der Gottesgläubigen und Kreationisten. Krauss sagt in dem oben verlinkten Interview:

    Fragesteller: But might it be possible to communicate the beauty of those discoveries without tacking on an assault on previous belief systems, especially when those belief systems aren’t necessarily scientific?
    &nbsp

    Krauss: Well, yes. I’m sympathetic to your point in one sense, and I’ve had this debate with Richard Dawkins; I’ve often said to him that if you want people to listen to you, the best way is not to go up to them and say, „You’re stupid.“ Somehow it doesn’t get through.

    It’s a fine line and it’s hard to tell where to fall on this one. What drove me to write this book was this discovery that the nature of „nothing“ had changed, that we’ve discovered that „nothing“ is almost everything and that it has properties. That to me is an amazing discovery. So how do I frame that? I frame it in terms of this question about something coming from nothing. And part of that is a reaction to these really pompous theologians who say, „out of nothing, nothing comes,“ because those are just empty words. I think at some point you need to provoke people. Science is meant to make people uncomfortable. And whether I went too far on one side or another of that line is an interesting question, but I suspect that if I can get people to be upset about that issue, then on some level I’ve raised awareness of it.

    Man muss das auch vor dem Hintergrund des US-amerikanischen Kreationismus sehen, der dort massiv in das Bildungssystem eingreift und viele Anhänger hat. Dem will Krauss offenbar etwas entgegensetzen.

    Du hättest auch noch mitnehmen sollen, dass es im Moment überhaupt keine anerkannte Definition für die „Gesamtenergie“ eines Universums gibt. Erst recht keine Definition einer „Gesamtenergie“, die dann mit der Expansion auch erhalten bleibt.

    Ich hatte schon aus vorherigen Diskussionen mit Bjoern und Dir mitgenommen, dass der Energieerhaltungssatz in der ART nicht bedingungslos gelten muss. Was mir allerdings irgendwie gegen den Strich geht, von so einem Prinzip verabschiedet man sich nicht gerne.

    Aber wenn man ehrlich ist, muss man zugeben, dass das einfach Geschmackssache ist. Unser momentanes Verständnis der Physik kann uns nur sagen, dass alle diese Möglichkeiten zutreffend sein könnten.

    Viele String-Theoretiker empfinden beispielsweise die String-Theorie als besonders elegant, während andere Quantengraviation-Theoretiker das ganz anders sehen.
    Eleganz ist immer etwas sehr Subjektives.

    Aus Äußerungen von Greene, Feynman und anderen habe ich mitgenommen, dass die korrekten Lösungen für physikalische Probleme meistens die „schönen“, „symmetrischen“ sind. Da passt das Nullenergie-Universum jedenfalls hinein. Am Anfang von Greenes Buch über String-Theorie dachte ich das auch von dieser, aber am Ende des Buchs fand ich die Verrenkungen, die dort unternommen werden mussten, um Dinge passend zu machen (z.B. dass die Grundschwingungen der Strings gar nicht zu den beobachteten Teilchen führen, und natürlich stapelweise extra Dimensionen) eher eine Karrikatur einer Lösung zu hinterlassen scheinen. Die String-Theorie ist mein Favorit nicht mehr.

    Aber wie Du sagst, das ist alles subjektiv, da ist alles noch offen. Aber gerade dann, wenn es mehrere Möglichkeiten gibt, hat man halt seine persönlichen Favoriten. Solange man weiß, dass diese nicht die einzige Möglichkeit sind, ist das m.E. in Ordnung.

    Und damit ab dafür 😉

  67. @Alderamin

    Na ja, aber die Frage nach dem Ursprung ist doch eines der zentralen Argumente der Gottesgläubigen und Kreationisten

    Das bleibt aber auch in diesem Modell offen. Dann stellt man eben die Frage nach dem Ursprung der Unschärferelation.
    Ein echtes „Nichts“ liegt nämlich wirklich nicht vor, da haben die „pompous theologians“ schon irgendwie recht.
    Der „God of the gaps“ ist unwiderlegbar.

    Aber ich versteh schon, worauf du hinaus willst.
    Ich ziehe man sollte Religion völlig aus der Kosmologie raushalten hiermit zurück.
    Nerven tuts mich aber trotzdem. 😉

    dass der Energieerhaltungssatz in der ART nicht bedingungslos gelten muss. Was mir allerdings irgendwie gegen den Strich geht

    Wobei es aber eigentlich überraschend wäre, wenn man sich von diesem Prinzip nicht verabschieden müsste.
    Dass Impuls, Drehimpuls und Energie Erhaltungsgrößen sind, lässt sich nämlich aus dem Noether-Theorem und damit jeweils aus bestimmten kontinuierlichen Symmetrien des Raumes herleiten.
    Diese Symmetrien sind also sozusagen die Ursache dafür, dass da etwas Erhalten bleibt.

    • Translationsinvarianz für Impuls (also Homogenität des Raumes)
    • Rotationsinvarianz für Drehimpuls (also Isotropie des Raumes)
    • Zeitinvarianz für die Energie (Homogenität der Zeit)

    Die Metrik eines expandierenden Friedmann-Universums ist allerdings im Allgmeinen zeitabhängig.
    https://upload.wikimedia.org/wikipedia/de/math/4/9/a/49ab35a518e8318bf54b2fc4dbdee8f6.png
    Deswegen bekommt man hier Probleme. Es ist dann offensichtlich nicht egal, welcher Zeitpunkt betrachtet wird. Die zugrunde liegende Symmetrie wird verletzt.

    Man kann es auch noch allgemeiner und ohne Rückgriff auf die FLRW-Metrik ausdrücken:
    Das Noether-Theorem ist für die ART nicht einsetzbar.
    Das liegt daran, dass die aus Symmetrien über das Noether-Theorem abgeleitete Energiedichte ausschließlich die an das „Gravitationsfeld“ koppelnde Materie und Felder beinhaltet und eben gerade nicht das „Gravitationsfeld“ selbst.
    (Wobei der Begriff „Gravitationsfeld“ ja eigentlich in der ART schon gar keine Bedeutung mehr hat. Müsste man also noch komplizierter formulieren. Ist fürs Grundverständnis aber wahrscheinlich nicht hilfreich, deswegen hab ichs gelassen.)

    Die Ursache bzw. das Hauptargument für die Existenz einer erhaltene Energie fällt also weg. Es gibt eigentlich keinen tieferen Grund mehr, so etwas für die ART zu erwarten und zu suchen.

    Aus Äußerungen von Greene, Feynman und anderen habe ich mitgenommen, dass die korrekten Lösungen für physikalische Probleme meistens die „schönen“, „symmetrischen“ sind.

    Unser Universum ist allerdings nicht immer schön und symmetrisch.

    Glücklicherweise.

    Das wir überhaupt existieren beweist zum Beispiel eine Asymmetrie zwischen Materie und Antimaterie.
    (Das beruht ja auf sehr tiefgehenden Verletzungen von Paritäts (P) – und Ladungs (C) – Symmetrien bzw. speziell auf der Verletzung der sogenannten CP-Symmetrie.)

    Die Beobachtung zeigt uns, dass auch noch eine weitere grundlegende Symmetrie grundlos verletzt wird.
    Das nennen wir den zweiten Hauptsatz der Thermodynamik, der der eine ausgezeichnete Richtung gibt.

    Und der LHC erledigt wahrscheinlich gerade still und heimlich die Supersymmetrie.
    https://www.scienceblogs.de/hier-wohnen-drachen/2011/03/wird-es-eng-fur-susy.php

    Da passt das Nullenergie-Universum jedenfalls hinein.

    Dass ein Nullenergie-Universum aber auch völlig anders als „aus dem Nichts heraus“ entstehen könnte ist klar. Oder?

    Aber wie Du sagst, das ist alles subjektiv, da ist alles noch offen. Aber gerade dann, wenn es mehrere Möglichkeiten gibt, hat man halt seine persönlichen Favoriten. Solange man weiß, dass diese nicht die einzige Möglichkeit sind, ist das m.E. in Ordnung.

    Richtig.

    Wenn dir aber fast alles, was ich geschrieben habe, schon vorher klar war, hab ich dich sehr stark missverstanden.
    Kam bei mir irgendwie ganz anders an. Tut mir leid.

  68. @Niels

    Wenn dir aber fast alles, was ich geschrieben habe, schon vorher klar war, hab ich dich sehr stark missverstanden.

    Na ja, alles nun auch wieder nicht, nur die wichtigsten Grundaussagen. Woher sollst Du auch wissen, was ich alles weiß oder auch nicht – ich weiß ja selbst zum Teil nicht mehr, was ich weiß oder nur zu wissen glaube, siehe die 25-Gramm-Geschichte oben 😉 Ich drücke mich manchmal vielleicht auch nicht klar genug aus, bis ja nur interessierter Laie.

    Jedenfalls wie immer danke für die ausführlichen Kommentare!!!

  69. Der letzte Post hier ist schon etwas her.
    Und zusätzlich bin ich mir nicht sicher, ob meine Frage schon beantwortet wurde.
    Die Antworten bei #8 und #9 habe ich gesehen, aber trotzdem passt da noch etwas nicht.

    Klar, man kann berechnen, wie lange man braucht um zur Singularität zu kommen.
    Allerdings dauert das für Außenstehende unendlich lange.
    Lauert da nicht irgendwo noch eine (mathematische) SIngularität?

    Gesetzt den Fall das Universum (oder das schwarze Loch) hat eine endliche Lebensdauer
    (big crunch, big rip, was auch immer): schafft es der Astronaut überhaupt bis zum Ereignishorizont?
    Oder holt ihn das Ende des Universums ein, oder er merkt, dass plötzlich das schwarze Loch weg ist
    (verstrahlt)?

    Noch etwas: der Ereignishorizont ist doch definiert als ‚Fluchtgeschwindigkeit = Lichtgeschwindigkeit).
    Kann man das rumdrehen zu: was ins schwarze Loch fällt hat am Ereignishorizont Lichtgeschwindigkeit?
    Das wäre ja verboten (masse unendlich, da haben wir vielleicht die SIngularität),
    Ist es möglich, den Ereignishorizont mit v < c zu durchqueren? Klingt komisch, weil dann könnte ich ja auch wieder aus dem schwarzen Loch entkommen.

    Wo ist denn da mein Denkfehler?

  70. @Myscibo

    Die Antwort #9 gilt aber immer noch:

    a) Für einen Außenstehenden sieht es so aus, als dauerte es unendlich lange, bis das Objekt den Ereignishorizont erreicht (d.h., es erreicht ihn nie). Dabei wird jedoch die Rotverschiebung unendlich groß, das Objekt wird in endlicher Zeit unsichtbar. Das liegt an der zunehmenden Zeitdilatation aufgrund des Schwerefelds, der das Licht unterliegt, das vom Rand des Schwarzen Lochs zum Beobachter unterwegs ist.

    b) Tatsächlich wird das Objekt aber gar nicht langsamer und die Strecke nicht kürzer, also ist das Objekt (je nach Größe des Schwarzen Lochs in Stunden (supermassives SL) bis Bruchteilen von Sekunden (stellares SL) in dessen Zentrum.

    c) Das Objekt wird beim Hineinfallen bis fast auf Lichtgeschwindigkeit beschleunigt und sieht deswegen die Strecke voraus stark verkürzt, deswegen würde es nach Borduhr noch viel schneller im Zentrum sein.

    Noch etwas: der Ereignishorizont ist doch definiert als ‘Fluchtgeschwindigkeit = Lichtgeschwindigkeit).
    Kann man das rumdrehen zu: was ins schwarze Loch fällt hat am Ereignishorizont Lichtgeschwindigkeit?

    Nein, kein massives Objekt kann die Lichtgeschwindigkeit erreichen, deshalb gilt dies Regel relativistisch gerechnet nicht mehr. Man wird nahe an die Lichtgeschwindigkeit beschleunigt, aber wegen der zunehmenden Masseträgheit wird man sie nicht erreichen.

    Ist es möglich, den Ereignishorizont mit v < c zu durchqueren? Klingt komisch, weil dann könnte ich ja auch wieder aus dem schwarzen Loch entkommen.

    Um das Schwarze Loch herum rotiert die gesamte Raumzeit, dem kann man sich nicht entgegenstemmen. Es reißt einen spiralförmig in die Tiefe, wenn man den Ereignishorizont erreicht. Außerhalb kann man noch knapp unterhalb der Lichtgeschwindigkeit einen Orbit halten.

  71. Ja, schon. Das hatte ich auch gesehen. Aber halt nicht geholfen.

    a) ist klar.

    Aber darf ich wegen a) auf das System von B umrechnen?
    Wenn B den Ereignishorizont erreicht, macht es SInn auf das System von a) (dem Außenstehenden) zurückzurechnen?
    Wahrscheinlich nicht, weil für den Außenstehenden unendlich viel Zeit vergangen ist.
    Frage: muss mich das stören?
    Offensichtlich nicht, aber das macht mir halt den Knoten ins Hirn.

    c) Der einfachheit halber rotiert die Raumzeit erst einmal nicht.
    Welche Geschwindigkeit hat also B wenn er den Ereignishorizont erreicht?
    B merkt selber nicht wann der Ereignishorizont erreicht ist,
    kann aber wahrscheinlich anhand seiner Zeit messen, wann es so weit sein sollte.

  72. @Myscibo

    Wenn B den Ereignishorizont erreicht, macht es SInn auf das System von a) (dem Außenstehenden) zurückzurechnen?

    Nicht wirklich: der Außenstehende sieht halt Stillstand am Ereignishorizont, der Hineinfallende nicht. Die Perspektive b) dient nur dazu klar zu machen, wie das zusammengehen kann, wird aber von niemandem beobachtet und ist so gesehen eigentlich irrelevant.

    c) Der einfachheit halber rotiert die Raumzeit erst einmal nicht.
    Welche Geschwindigkeit hat also B wenn er den Ereignishorizont erreicht?

    Ich hab‘ in einem Forum folgendes gefunden (ohne Herleitung): 2*3^-(3/2)*c (aber ist das die Perspektive b) oder c)?)

    Guck‘ mal hier, S. 64, vielleicht hilft Dir das weiter (ich hab‘ jetzt nicht wirklich Zeit, mich da reinzugraben)

  73. @Myscibco
    Hast du meinen Beitrag #25 gesehen? An anderer Stelle habe ich das nochmal minimal anders formuliert, dort habe ich außerdem zusätzlich auch noch etwas zur „Singularität“ beim Schwarzschildradius geschrieben.

    @Alderamin @Myscibco
    Dieses 2*3^-(3/2)*c hat nichts mit dem Schwarzschildradius zu tun.

    Wir betrachten einen weit entfernten Beobachter, also a).
    Ein bei ihm mit der Geschwindigkeit Null startendes Testteilchen fällt in das Schwarze Loch.
    Wir betrachten mathematisch also der Einfachheit halber einen freien Fall in radialer Richtung aus dem Unendlichen. (Radiale Richtung bedeutet, dass sich in Kugelkoordinaten nur die Koordinate R ändert.)

    Man kann zeigen, dass der Beobachter a) (der stationär bleibt, also nicht ins Loch fällt) für das frei fallende Testteilchen die Geschwindigkeit
    v = c * (1-Rs/R) * [(Rs/R)^(1/2)]
    (Rs ist die Schwarzschildradius, R der Abstand zum Zentrum des schwarzen Loches)
    wahrnimmt.
    Wenn man das ableitet, bemerkt man, dass es ein Extremum der Geschwindigkeit gibt, nämlich bei R = 3 Rs. Dort hat die Geschwindigkeit ihren höchsten Wert, nämlich
    v = (1-1/3)*(1/3)^(1/2)*c = (4/27)^(1/2)*c = 2*3^-(3/2)*c.
    Für diesen Beobachter a) fällt das Teilchen also mit zunehmender Geschwindigkeit, bis diese bei R = 3 Rs ihr Maximum erreicht. Anschließend wird es wieder langsamer.
    Wenn man sich die Formel vor die Geschwindigkeit ansieht, dauert die Annäherung an den Schwarzschildradius R = Rs unendlich lange, die Geschwindigkeit geht bei Annäherung an den Schwarzschildradius gegen Null.

    Ein anderer Beobachter d), der sich beispielsweise bei R = Rs + 1m befindet, der also einen Meter über dem Schwarzschildradius ruht, sieht dieses Testteilchen dagegen mit beinahe Lichtgeschwindigkeit an sich vorbeifliegen. (Das ergibt sich aus einer anderen Formel, die aber so kompliziert ist, dass ich sieh nur ungern abtippen will.)
    Ein Beobachter e), der sich nur einen halben Meter über dem Schwarzschildradius befindet, sieht das Testteilchen mit einer Geschwindigkeit noch ein bisschen näher an der Lichtgeschwindigkeit an sich vorbeifliegen.
    Und so weiter.
    (Einen massebehafteten Beobachter, der exakt am Ereignishorizont ruht, kann es dagegen prinzipiell nicht geben.)

    Für in mittlerer Entfernung ruhende Beobachter gibt es dann eine noch kompliziertere Formel, aus der sich die Formel für a) sowie die gemeinsame Formel für d) und e) jeweils durch Grenzwertbetrachtungen gewinnen lassen.

    (Das alles gilt jetzt nur für ein Schwarzschild-Scharze-Löcher. Ein rotierendes Kerr-Loch tue ich mir nur unter Zwang an, da stoße nämlich ich da bei der Ableitung der Formeln an meine Grenzen und es wird anstrengend und zeitaufwendig.
    Das würde außerdem wahrscheinlich sowieso nur verwirren.

  74. @Niels

    Cool, und aus Sicht des fallenden Beobachters, würde der am Schwartzschildradius aus Symmetriegründen das SL mit dem gleichen Tempo näher kommen sehen, wie der Beobachter auf gleichem Radius ihn?

    Und wenn er durch den Schwarzschild-Radius hindurch fiele, könnte er ja auch nicht schneller als c werden, andererseits sähe er dann wohl nichts mehr, weil ihm keine Strahlung mehr entgegen käme, und es ginge ihm somit der Bezugspunkt für eine Geschwindigkeitsmessung verloren.

    Abgesehen davon, dass durch die Längenverkürzung das SL eh‘ zu einer platten Scheibe zusammengedrückt erschiene und die Spaghettifizierung nach Bordzeit noch viel schneller käme, als Rs/c (bei einem supermassiven SL, sonst käme sie schon draußen).

  75. @Alderamin: danke für den Link.
    Uff, schwere Kost. Mal sehen …

    @Niels: danke für den Link. Und für die Erklärung. Das mit der Koordinatensingularität muss ich erst einmal setzen lassen.
    Weil ich so ganz das noch nicht raus habe.
    Beobachter A misst eine unendliche Zeit, oder?
    Er sieht ja zu jeder seiner beliebigen Zeit wie B sich bewegt.
    Wenn er immer noch B sieht während der big rip (wenn es den gibt …) ihn erwischt,
    dann muss das doch auch B erwischen. Darf man von A auf B’s Eigenzeit transformieren? Oder besser: von B auf A, nachdem B den Ereignishorizont durchquert hat?

    Egal, ich lese mal in die Links rein. Danke.

  76. Zur Spaghettisierung:
    Ich habe gerade einen interessanten Artikel in den SPektrum highlights gelesen,
    dass es auch ohne Ereignishorizont SIngularitäten geben kann. Die würde ich für meinen Versuch mit den Raumschiffen wählen,
    weil ich ja auch Ergebnisse sehen will …

  77. @Alderamin
    Wir hatten es vor kurzem schon einmal mit der Zeitmessung in der ART.
    In einer Schwarzschild-Raumzeit gibt es keine globale Zeit mehr. Für Beobachter mit unterschiedlichem Abstand R zur Singularität vergeht die Zeit also unterschiedlich. Es gilt also nur noch Kugelsymmetrie.

    Wir betrachten jetzt zwei Beobachter.
    Beobachter A ruht mit seinem Raumschiff bei einem festen Abstand R. Das bedeutet, dass sein Antrieb eingeschaltet sein muss.
    Beobachter B fällt ins schwarze Loch, der Abstand R ändert sich also laufend. Sein Antrieb ist abgeschaltet, er befindet sich im freien Fall.
    Die spezielle Relativitätstheorie gilt in der ART nur noch lokal, ganz genau genommen also nur in einem einzigen Raumzeitpunkt. Umso stärker die Raumkrümmung, umso genauer muss man es nehmen.
    Nur in dem Moment, in dem Beobachter B Beobachter A passiert, befinden sich A und B auf demselben Abstand R. A sieht B also mit derselben Geschwindigkeit vorbeifliegen, in der auch Beobachter B Beobachter A vorbeifliegen sieht.
    Für alle anderen Abstände messen A und B ganz andere Zeiten und damit natürlich auch ganz unterschiedliche Geschwindigkeiten und Geschwindigkeitsveränderungen.

    Ich probiere es mal mit einem Beispiel zu verdeutlichen:
    Beobachter C ruht relativ nahe (aber außerhalb) des Schwarzschildradius, Beobachter D ruht sehr weit vom schwarzen Loch entfernt.
    Beobachter C sendet ein Photon senkrecht zum Schwerefeld aus, das nach kurzer Entfernung an einem Spiegel reflektiert wird und dadurch wieder zu C zurückkehrt.
    Beobachter D misst eine viel längere Laufzeit als Beobachter C, für ihn war das Photon also mit einer Geschwindigkeit deutlich unterhalb der Lichtgeschwindigkeit unterwegs.
    Jetzt kehren wir das Ganze um. D sendet das Photon aus und lässt es zurückreflektieren. Für C hat sich das Photon mit deutlicher Überlichtgeschwindigkeit bewegt.

    Cool, und aus Sicht des fallenden Beobachters, würde der am Schwartzschildradius aus Symmetriegründen das SL mit dem gleichen Tempo näher kommen sehen, wie der Beobachter auf gleichem Radius ihn?

    Siehe oben. Das habe ich geschrieben, weil ich die Frage nicht ganz verstehe.
    Da geht es dir darum, wie der frei fallende Beobachter die Singularität näherkommen sieht? Natürlich gar nicht, es können ihn doch gar keine Photonen von innen erreichen?

    Und wenn er durch den Schwarzschild-Radius hindurch fiele, könnte er ja auch nicht schneller als c werden

    Welchen Beobachter misst die Geschwindigkeit?
    (Er würde jedenfalls immer mit langsamerer Geschwindigkeit als der eines Photons beobachtet.)
    Der frei fallende Beobachter hat eine Eigenzeit, damit kann man natürlich auch eine Eigengeschwindigkeit berechnen.
    Die ist natürlich kleiner als die Lichtgeschwindigkeit, allerdings ist das keine Geschwindigkeit, die irgendjemand irgendwie messen könnte.

    @Myscibco
    Betrachten wir mal folgendes Beispiel:
    Zwei Astronauten (Alice und Bob) befinden sich in einem Raumschiff auf einer stationären Umlaufbahn um ein schwarzes Loch. (Natürlich oberhalb das Schwarschildradius.)
    Alice nimmt Reparaturen an der Außenhülle des Raumschiffs vor und drückt dabei aus Versehen einen Knopf, der die Antriebsdüse ihres Raumanzuges anspringen lässt.
    Dadurch wird sie vom Raumschiff abgetrieben und beginnt den freien Fall ins schwarze Loch. Die Uhr an ihrem Handgelenk misst natürlich ihre Eigenzeit, nach der endlichen Zeit Ts hat sie die Singularität erreicht.

    Wann muss Bob offiziell ihren Tod feststellen?
    Gar nicht, weil Alice aus seiner Sicht den Schwarzschild-Radius erst nach unendlich langer Zeit überquert? Nach der Zeit Ts?

    Ich hab mal irgendwo folgende Antwort gelesen, die fand ich ganz stimmig:
    „Alice ist am Leben“ hat nur dann eine praktische Bedeutung, solange Bob Alice noch retten kann. Seine Rettungsmission muss er zwingend unternehmen, bevor die Zeit Tr auf seiner Armbanduhr abgelaufen ist.
    Tr ist dabei der Zeitpunkt, zu dem ein vom Raumschiff abgestrahltes Photon Alice in genau dem Moment erreicht, in dem sie aus ihrer Sicht den Schwarzschild-Radius überschreitet.
    Wenn Bob seine Rettungsaktion also nicht startet, bevor Tr vergangen ist, kann er Alice nicht mehr vor dem Schwarzschild-Radius abfangen. (Nicht einmal Licht kann das.)
    Er kann Tr also zu ihrem Todeszeitpunkt erklären.

    Die Sicht eines Bobs, der einfach an Ort und Stelle bleibt, kann man mit der oben beschriebenen Situation vergleichen, bei der C bei Photonen Überlichtgeschwindigkeit beobachtet. Das ist eigentlich kein „reales“ Ereignis am Ort des Geschehens.
    Durch die zwischen Bob und Alice verlaufende Raumzeitkrümmung („durch das dazwischen liegenden Gravitationsfeld“) nimmt Bob also sozusagen nur Teile von Alices “Realität“ wahr.
    Alles, was Bob nach dem Vergehen des Zeit Tr noch von Alice wahrnimmt, ist gravitativ verzögerte Information von längst vergangenem Geschehen.

    Werden wir zur Verdeutlichung mal romantisch:
    Alice kann in ihrer Eigenzeit Ts gerade noch die Botschaft „Bob, ich liebe dich“ senden.
    Bob empfängt das „Bo“ sehr schnell, dann wird das ganze unglaublich verzerrt und in die Länge gezogen. Das „i“ dauert 2 Minuten, „c“ mehrere Monate, …
    Aber auch wenn Bob unendlich lange wartet, kann er nur „Bob, ich“ empfangen.
    Den Rest hat Alice nämlich erst nach Überschreiten des Schwarzschild-Radius ausgesprochen.
    (Spaghettifizierung vernachlässigen wir bei diesem Beispiel.
    In der Praxis werden die Radio-Signale außerdem auch noch rotverschoben, nach dem „Bo“ kann man also sowieso nix mehr empfangen, selbst wenn man unendlich lange wartet. Die Rotverschiebung geht nämlich ebenfalls gegen unendlich.)

    Zur Spaghettisierung:
    Ich habe gerade einen interessanten Artikel in den SPektrum highlights gelesen,
    dass es auch ohne Ereignishorizont SIngularitäten geben kann

    Die Spaghettisierung hat aber gar nichts mit dem Ereignishorizont zu tun? Dazu braucht man nur starke Raumkrümmung.

  78. Hm, da war meine Formulierung vielleicht nicht ganz klar.

    Tr ist dabei der Zeitpunkt, zu dem ein vom Raumschiff abgestrahltes Photon Alice in genau dem Moment erreicht, in dem sie aus ihrer Sicht den Schwarzschild-Radius überschreitet.

    Das Photon wird zum Zeitpunkt Tr abgestrahlt.
    Das ist der letzte Zeitpunkt, bei dem (vom ruhenden Raumschiff) abgestrahlte Photonen Alice noch vor der Überquerung des Ereignishorizontes erreichen können.
    Ein Raumschiff kann natürlich nicht schneller als mit c fliegen, genau genommen kann es c nicht einmal ganz erreichen.
    Deswegen muss Bob zwingend vor dem Zeitpunkt Tr starten.

  79. Alles was man heute über „Schwarze Löcher“ weiß, ist auf dem ungefähren Wissenssttand des Mittelalters zu der Erdkugel! Niemand weiß wirklich was passieren würde, wenn man in ein schwarzes Loch gerät. Etwas Beständiges kann sich nicht in „nichts“ auflösen“. Selbst Steven Hawking wird uns nicht die Antwort bieten können, welche hinter dem Mysterium „Schwarzes Loch“ steckt. Nur eine direkte Reise in eines dieser Löcher würde die Frage beantworten um was es sich dabei genau handelt. Bis die Menscheit aber die Möglichkeiten dazu hat, werden noch ein paar Hunderte Jahre vergehen und alles wird bis dahin einfach nur Theorie sein, welche sich selbst heute aus logischer Sicht als Humbug erweisen wird. Alles was in Lehrbüchern geschrieben steht und alles was hier über schwarze Löcher diskutiert wird, gehört ebenfalls zu diesem Humbug, da keiner die genaue Antwort darauf kennt!

    1. @Unbekannt: „Alles was in Lehrbüchern geschrieben steht und alles was hier über schwarze Löcher diskutiert wird, gehört ebenfalls zu diesem Humbug,“

      Die gesamte astronomische Forschung/Beobachtung zu schwarzen Löcher ist dann also gefälscht? Erfunden? Gibts für diese Behauptung auch Belege?

  80. @Unbekannt

    Etwas Beständiges kann sich nicht in “nichts” auflösen”.

    Tut es ja nicht, die Masse ist ja noch da. Das erste nachgewiesene Schwarze Loch, Cygnus X-1, ist nicht nur eine Quelle von Röntgenstrahlung (die nicht aus dem Schwarzen Loch stammt, sondern von dem aufgeheizten Gas, das vor dem Absturz um es herumjagt), sondern lässt auch seinen Begleitstern mächtig herumeiern. Da ist also etwas sehr massives, nur kein Stern. Gleiches gilt für Sagittarius A* im Zentrum der Milchstraße. Da kreisen andere Sterne mit irrsinnigem Tempo um ein paar Millionen Sonnenmassen (nur so können sie bei dieser Geschwindigkeit ihre Bahn einhalten), von denen man nichts sieht. Nur ein bisschen Radiostrahlung. Hast Du eine alternative Erklärung dafür, was sich da befinden soll?

    Bis die Menscheit aber die Möglichkeiten dazu hat, werden noch ein paar Hunderte Jahre vergehen und alles wird bis dahin einfach nur Theorie sein,

    Besseres alsTheorien gibt es in der Wissenschaft nicht. Die Kugelgestalt der Erde ist auch nur eine wissenschaftliche Theorie.

    Schon bald werden wir übrigens ein Schwarzes Loch direkt sehen können: https://www.mpifr-bonn.mpg.de/mitteilungen/2015/6

  81. @Unbekannt

    „Selbst Steven Hawking wird uns nicht die Antwort bieten können“
    Muss er ja auch nicht. Verlangt keiner von ihm. Das bedeutet aber nicht, dass es unmöglich ist etwas über SL zu erfahren.

    „diesem Humbug, da keiner die genaue Antwort darauf kennt!“
    Deswegen kann man aber trotzdem darüber reden. Sollte man sogar…..

Schreibe einen Kommentar zu Niels Antworten abbrechen

Deine E-Mail-Adresse wird nicht veröffentlicht. Erforderliche Felder sind mit * markiert

Diese Website verwendet Akismet, um Spam zu reduzieren. Erfahre mehr darüber, wie deine Kommentardaten verarbeitet werden.